Re: [obm-l] Re: [obm-l] equação do 2 grau

2018-06-16 Por tôpico gugu
   Bem, sobrou o caso a=b=0... Mas eu não gosto muito do enunciado -  
eu escreveria "...pelo menos uma raiz REAL comum" - de fato, se a=b  
então as equações têm raízes complexas comuns.

   Abraços,
     Gugu

Quoting Pedro José :


Boa noite!
Como é uma questão de múltipla escolha, dá para perceber uma restrição
quanto ao|R.
Se a<>b. Se o delta de uma das equações for >= 0, o outro será menor que 0.
Portanto não há soluções.
Saudações,
PJMS

Em Sáb, 16 de jun de 2018 16:59, luciano rodrigues 
escreveu:


Se a=b então o delta é negativo.

> Em 16 de jun de 2018, às 16:09, Daniel Quevedo 
escreveu:
>
> O número de pares ordenados (a, b), de números reais tais que as
equações x^2 + ax + b^2 = 0 e x^2 + bx + a^2 = 0 possuem pelo menos uma
raiz comum é:
> a) 0
> b) 1
> c) 2
> d) 3
> e) 4
>
> R: 0
>
> PS: Não entendi a questão pq, se a = b, as equações são iguais e
assim satisfarão a condição (pelo menos uma raiz comum), mesmo que essas
não sejam reais. Mas como provar que para a diferente de b não há raizes
comuns?
>
>
> --
> Esta mensagem foi verificada pelo sistema de antivírus e
> acredita-se estar livre de perigo.

--
Esta mensagem foi verificada pelo sistema de antivírus e
 acredita-se estar livre de perigo.


=
Instru�ões para entrar na lista, sair da lista e usar a lista em
http://www.mat.puc-rio.br/~obmlistas/obm-l.html
=



--
Esta mensagem foi verificada pelo sistema de antiv?rus e
 acredita-se estar livre de perigo.





--
Esta mensagem foi verificada pelo sistema de antiv�rus e
acredita-se estar livre de perigo.

=
Instru��es para entrar na lista, sair da lista e usar a lista em
http://www.mat.puc-rio.br/~obmlistas/obm-l.html
=


Re: [obm-l] Soma (k = 1, n) 1/P'(r_k) = 0

2018-04-16 Por tôpico gugu
   Dá para fazer com interpolação de Lagrange: o único polinômio de  
grau <=n-1 que vale 1 em r_i para 1<=i<=n (que obviamente é o  
polinômio constante igual a 1) é dado por
soma_(1<=i<=n)Produto_(j<>i)((x-r_j)/(r_i-r_j)). Aí é só olhar para o  
coeficiente de x^[n-1} (que é 0).

   Abraços,
 Gugu

Quoting Claudio Buffara <claudio.buff...@gmail.com>:


Resumo da ópera: ainda não temos uma demonstração elementar disso.

Mas não deixa de ser interessante tentar dar uma interpretação geométrica
da expressão para polinômios de grau baixo que tenham todas as raízes reais
e distintas.

Grau 2 é meio evidente:  as retas tangentes à parábola nas raízes têm
inclinações de mesma magnitude e sinais opostos.

Grau 3 é mais interessante...
De cara, dá margem ao problema: "Uma função polinomial de grau 3 tem raízes
(ou será que é melhor chamar de "zeros" - funções têm zeros, quem tem
raízes são equações...) reais e distintas: a, b e c. Se as retas tangentes
ao gráfico da função nos pontos (a,0) e (b,0) têm inclinações m e n,
respectivamente, determine a inclinação da reta tangente ao gráfico no
ponto (c,0).", cuja tentativa mais óbvia de solução vai descambar num monte
de contas (desnecessárias) envolvendo a, b e c, quando a resposta é
-mn/(m+n).

[]s,
Claudio.

2018-04-16 14:10 GMT-03:00 Douglas Oliveira de Lima <
profdouglaso.del...@gmail.com>:


Entao a questao é até que ponto ela é verdadeira , pois funciona para
casos elementares.

Douglas Oliveira

Em dom, 15 de abr de 2018 22:29, Bernardo Freitas Paulo da Costa <
bernardo...@gmail.com> escreveu:


2018-04-15 13:09 GMT-03:00 Douglas Oliveira de Lima
<profdouglaso.del...@gmail.com>:
> Usa o polinomio de Lagrange , nao é nada obvia mesmo.

Como usa Lagrange, a fórmula segue para k = 0, 1, ... n-1
(interpolando em n pontos, vamos até grau n-1).  E é, de fato, falso
para k = n, use P(x) = (x-1)(x+1).

Além disso, mesmo para k = n-1, a demonstração por complexa não se
aplica mais (o grau dá errado...), e o mesmo polinômio serve para
mostrar que a soma não dá mais zero.

Abraços,
--
Bernardo Freitas Paulo da Costa

--
Esta mensagem foi verificada pelo sistema de antivírus e
 acredita-se estar livre de perigo.


=
Instru�ões para entrar na lista, sair da lista e usar a lista em
http://www.mat.puc-rio.br/~obmlistas/obm-l.html
=



--
Esta mensagem foi verificada pelo sistema de antivírus e
acredita-se estar livre de perigo.



--
Esta mensagem foi verificada pelo sistema de antiv?rus e
 acredita-se estar livre de perigo.





--
Esta mensagem foi verificada pelo sistema de antiv�rus e
acredita-se estar livre de perigo.

=
Instru��es para entrar na lista, sair da lista e usar a lista em
http://www.mat.puc-rio.br/~obmlistas/obm-l.html
=


Re: [obm-l] Re: Probleminha um tanto estranho

2018-04-10 Por tôpico gugu

   Oi Claudio,
   Mais ou menos: se a=3, b=4 e c=5, sua afirmação diz que um  
polinômio em Z[x] que tenha (3+4i)/5 como raiz deve ser divisível em  
Z[x] por 25z^2-30z+25, mas poderia ser 5z^2-6z+5. Mas se mdc(a,b,c)=1  
e 2|c^2*z^2 - 2ac*z + (a^2+b^2), devemos ter c par e a e b ímpares,  
donde a^2+b^2=2 (mod 4), e só podemos tirar um fator 2, ficando o  
coeficiente ac de z ainda par - assim, a afirmação do Artur para  
polinômios quadráticos continua provada.

   Abraços,
 Gugu

Quoting Claudio Buffara <claudio.buff...@gmail.com>:


Se um polinômio com coeficientes inteiros tiver (a+bi)/c como raiz (a,b,c
inteiros), então também terá (a-bi)/c.
Assim, será divisível por f(z) = c^2*z^2 - 2ac*z + (a^2+b^2)
(incidentalmente, isso prova a sua afirmação para polinômios quadráticos:
2ac é necessariamente par).

f(z) | 37971 z^998  + ... + 67917  ==> a^2+b^2 divide 67917 = 3*22639. Mas
3 e 22639 são primos da forma 4k+3. Logo, 67917 não é divisível pela soma
de dois quadrados.

[]s,
Claudio.



2018-04-09 10:54 GMT-03:00 Artur Steiner <artur.costa.stei...@gmail.com>:


Isto é uma generalização do seguinte fato: Se todos os coeficientes de  um
pol. do 2o grau forem ímpares, então o pol. não apresenta nenhuma raiz com
ambas as partes racionais.

Artur Costa Steiner

Em Dom, 8 de abr de 2018 19:56, Artur Steiner <
artur.costa.stei...@gmail.com> escreveu:


Mostre que o polinômio

P(x) = 37971 x^998 - 74914 x^721 - 8677 x^432 + 12674 x^297 -  21438
x^129 + 67917

não tem nenhuma raiz com ambas as partes racionais

Abraços.

Artur Costa Steiner



--
Esta mensagem foi verificada pelo sistema de antivírus e
acredita-se estar livre de perigo.



--
Esta mensagem foi verificada pelo sistema de antiv?rus e
 acredita-se estar livre de perigo.





--
Esta mensagem foi verificada pelo sistema de antiv�rus e
acredita-se estar livre de perigo.

=
Instru��es para entrar na lista, sair da lista e usar a lista em
http://www.mat.puc-rio.br/~obmlistas/obm-l.html
=


Re: [obm-l] Re: [obm-l] Re: [obm-l] Re: [obm-l] Teoria dos números

2018-03-22 Por tôpico gugu

   Oi Claudio,
   Eu não sei de onde veio a substituição mágica do Anderson Torres -  
só achei uma fatoração na expressão obtida a partir dela... Não sou  
especialmente fã desse tipo de problema.

   Abraços,
 Gugu

Quoting Claudio Buffara <claudio.buff...@gmail.com>:


Tudo muito bom, mas o que ninguém explicou é como foram obtidas as
fatorações/transformações algébricas mágicas.
Insight?
Conhecimentos prévios?
Tentativa e erro e muito braço?

[]s,
Claudio.


2018-03-21 18:54 GMT-03:00 <g...@impa.br>:


Sim, e fazendo a=u/2, b=v/2 e c=w/2 temos (u+v+w)(uv+uw+vw)-uvw=2, ou
seja, u^2v+uv^2+u^2w+uw^2+v^2w+vw^2+2uvw=2, mas
u^2v+uv^2+u^2w+uw^2+v^2w+vw^2+2uvw=(u+v)(u+w)(v+w). Assim, podemos ter
u+v=2, u+w=v+w=1, o que dá w=0, u=v=1; u+v=2, u+w=v+w=-1, o que dá w=-2,
u=v=1; u+v=-2, u+w=1, v+w=-1, o que dá w=1, u=0, v=-2; u+v=-2, u+w=-1,
v+w=1, o que dá w=1, u=-2, v=0 e as coisas simétricas. Daí saem as soluções
(1,0,0), (2,-1,-1), (-3/2,-1/2,3/2), (-3/2,3/2,-1/2) (e as coisas
simétricas), mas as duas últimas não são inteiras, de modo que só temos as
soluções que já tínhamos achado...
   Abraços,
 Gugu

Quoting Bernardo Freitas Paulo da Costa <bernardo...@gmail.com>:

2018-03-20 23:14 GMT-03:00 Anderson Torres <torres.anderson...@gmail.com>:



Em 13 de março de 2018 20:19, Douglas Oliveira de Lima
<profdouglaso.del...@gmail.com> escreveu:


Essa achei legal e estou postando.

Resolva nos inteiros a seguinte equação:  (x + y)(y + z)(z + x)/2 + (x
+ y +
z)3 = 1 – xyz .



Substituição mágica: x=-a+b+c, y=a-b+c, z=a+b-c. Com isso, x+y=2c,
x+y+z=a+b+c e

4abc + (a+b+c)^3 + (-a+b+c)(a-b+c)(a+b-c) = 1

Usando polinômios simétricos,

4(a+b+c)(ab+ac+bc) - 4abc = 1

Agora estou confuso...



Note que a,b,c não precisam mais ser inteiros, podem ser inteiros
divididos por 2 (se não me engano)

Abraços,
--
Bernardo Freitas Paulo da Costa

--
Esta mensagem foi verificada pelo sistema de antiv?rus e
 acredita-se estar livre de perigo.


=
Instru??es para entrar na lista, sair da lista e usar a lista em
http://www.mat.puc-rio.br/~obmlistas/obm-l.html
=






--
Esta mensagem foi verificada pelo sistema de antivírus e
acredita-se estar livre de perigo.

=
Instru�ões para entrar na lista, sair da lista e usar a lista em
http://www.mat.puc-rio.br/~obmlistas/obm-l.html
=



--
Esta mensagem foi verificada pelo sistema de antiv?rus e
 acredita-se estar livre de perigo.





--
Esta mensagem foi verificada pelo sistema de antiv�rus e
acredita-se estar livre de perigo.

=
Instru��es para entrar na lista, sair da lista e usar a lista em
http://www.mat.puc-rio.br/~obmlistas/obm-l.html
=


Re: [obm-l] Re: [obm-l] Re: [obm-l] Teoria dos números

2018-03-21 Por tôpico gugu
Sim, e fazendo a=u/2, b=v/2 e c=w/2 temos (u+v+w)(uv+uw+vw)-uvw=2, ou  
seja, u^2v+uv^2+u^2w+uw^2+v^2w+vw^2+2uvw=2, mas  
u^2v+uv^2+u^2w+uw^2+v^2w+vw^2+2uvw=(u+v)(u+w)(v+w). Assim, podemos ter  
u+v=2, u+w=v+w=1, o que dá w=0, u=v=1; u+v=2, u+w=v+w=-1, o que dá  
w=-2, u=v=1; u+v=-2, u+w=1, v+w=-1, o que dá w=1, u=0, v=-2; u+v=-2,  
u+w=-1, v+w=1, o que dá w=1, u=-2, v=0 e as coisas simétricas. Daí  
saem as soluções (1,0,0), (2,-1,-1), (-3/2,-1/2,3/2), (-3/2,3/2,-1/2)  
(e as coisas simétricas), mas as duas últimas não são inteiras, de  
modo que só temos as soluções que já tínhamos achado...

   Abraços,
 Gugu

Quoting Bernardo Freitas Paulo da Costa <bernardo...@gmail.com>:


2018-03-20 23:14 GMT-03:00 Anderson Torres <torres.anderson...@gmail.com>:

Em 13 de março de 2018 20:19, Douglas Oliveira de Lima
<profdouglaso.del...@gmail.com> escreveu:

Essa achei legal e estou postando.

Resolva nos inteiros a seguinte equação:  (x + y)(y + z)(z + x)/2  
+ (x + y +

z)3 = 1 – xyz .



Substituição mágica: x=-a+b+c, y=a-b+c, z=a+b-c. Com isso, x+y=2c,  
x+y+z=a+b+c e


4abc + (a+b+c)^3 + (-a+b+c)(a-b+c)(a+b-c) = 1

Usando polinômios simétricos,

4(a+b+c)(ab+ac+bc) - 4abc = 1

Agora estou confuso...


Note que a,b,c não precisam mais ser inteiros, podem ser inteiros
divididos por 2 (se não me engano)

Abraços,
--
Bernardo Freitas Paulo da Costa

--
Esta mensagem foi verificada pelo sistema de antiv?rus e
 acredita-se estar livre de perigo.


=
Instru??es para entrar na lista, sair da lista e usar a lista em
http://www.mat.puc-rio.br/~obmlistas/obm-l.html
=





--
Esta mensagem foi verificada pelo sistema de antiv�rus e
acredita-se estar livre de perigo.

=
Instru��es para entrar na lista, sair da lista e usar a lista em
http://www.mat.puc-rio.br/~obmlistas/obm-l.html
=


Re: [obm-l] Frações contínuas

2017-11-08 Por tôpico gugu

Caro Israel,
Toda fração contínua infinita cujos coeficientes são inteiros  
positivos (não funções...) é irracional.

Abraços,
  Gugu

Quoting Israel Meireles Chrisostomo <israelmchrisost...@gmail.com>:


Olá pessoal, eu li recentemente que toda fração contínua infinita é
irracional.Vejam essa fração contínua abaixo
[image: Imagem inline 1]

Se eu substituir x por pi/2 eu vou obter zero no lado esquerdo, mas a
fração contínua é infinita pois seus convergentes nunca se anulam.Alguém
poderia me explicar isso?
--
Israel Meireles Chrisostomo

--
Esta mensagem foi verificada pelo sistema de antiv?rus e
 acredita-se estar livre de perigo.





--
Esta mensagem foi verificada pelo sistema de antiv�rus e
acredita-se estar livre de perigo.

=
Instru��es para entrar na lista, sair da lista e usar a lista em
http://www.mat.puc-rio.br/~obmlistas/obm-l.html
=


Re: [obm-l] Mostrar que esta integral complexa é um número inteiro

2016-12-07 Por tôpico gugu

   Cara Ana,
   Do jeito que está enunciado creio que não é verdade: se f tem dois  
zeros simples p e q em V e S é uma curva com a forma do número 8 que  
dá uma volta no sentido anti-horário em torno de p e uma volta no  
sentido horário em torno de q (i.e., Ind(S,p)=1 e Ind(S,q)=-1), então  
o número de zeros de f em {z em V | Ind(S, z) > 0)} é 1 mas 1/(2pi i)  
Integral(sobre S) (f'(z)/f(z)) dz vale 1-1=0.
   Em geral, 1/(2pi i) Integral(sobre S) (f'(z)/f(z)) dz é igual à  
soma sobre os zeros w de f em V de Ind(S,w).mult(f,w), onde mult(f,w)  
é a multiplicidade do zero w de f. Se não há nenhum zero w de f com  
Ind(S,w)<0 então a sua conclusão está correta.

   Abraços,
     Gugu

Quoting "(null) (null)" <ana...@yahoo.com>:


Oi amigos!

Gostaria de uma ajuda nisso:

Seja V um subconjunto aberto e conexo do plano complexo C e seja S   
uma curva suave e fechada em V tal que que Ind(S,z) = 0 para todo z   
em C/V. Seja f uma função holomorfa em V que não apresente nenhum   
zero no traço S* de S.


Mostre que 1/(2pi i) Integral(sobre S) (f'(z)/f(z)) dz = M, onde M é  
 um inteiro tal que M >= número de zeros de f em {z em V | Ind(S, z)  
 > 0)} (contando multiplicidades).


Muito obrigada.

Ana

--
Esta mensagem foi verificada pelo sistema de antiv�rus e
 acredita-se estar livre de perigo.


=
Instru��es para entrar na lista, sair da lista e usar a lista em
http://www.mat.puc-rio.br/~obmlistas/obm-l.html
=







This message was sent using IMP, the Internet Messaging Program.



--
Esta mensagem foi verificada pelo sistema de antiv�rus e
acredita-se estar livre de perigo.


=
Instru��es para entrar na lista, sair da lista e usar a lista em
http://www.mat.puc-rio.br/~obmlistas/obm-l.html
=


Re: [obm-l] Re: [obm-l] Re: [obm-l] Polinômio irredutível em Z

2016-11-23 Por tôpico gugu

   Oi pessoal,
   Na solução do link os coeficientes do polinômio são primos, e numa  
fatoração qualquer um dos fatores vai ser mônico (a menos de sinal),  
donde o produto dos módulos de suas raízes será pelo menos 1, uma  
contradição se todas as raízes têm módulo menor que 1.

   Abraços,
 Gugu

Quoting Bernardo Freitas Paulo da Costa <bernardo...@gmail.com>:


2016-11-23 14:21 GMT-02:00 Anderson Torres <torres.anderson...@gmail.com>:

Isso não me parece verdadeiro - (2x-1)^1000 é um contraexemplo.


Acho que tem uma hipótese implícita de que todas as raízes são distintas.

Abraços,
--
Bernardo Freitas Paulo da Costa


Em 13 de novembro de 2016 14:20, Adrian Alexander Delgado
<adrian.alexander4...@gmail.com> escreveu:

É sobre esse problema:
(Irã 2007) Existe uma sequência de inteiros a_0, a_1, a_2, ... tais que
(a_i,a_j)=1 para i diferente de j e para todo n inteiro positivo   
a_0 + a_1 x

+... +a_n x^n é irredutível em Z[x]?

No fórum AoPS, vi que a solução usa o fato de que
Se toda raiz complexa ? de f satisfaz |?|<1, então f é irredutível em Z

Tentei procura uma demonstração disso na internet e não encontrei.
Alguém sabe como demonstrar isso?

Link da solução:
http://artofproblemsolving.com/community/c6h149740p847418



--
Esta mensagem foi verificada pelo sistema de antiv?rus e
 acredita-se estar livre de perigo.


=
Instru?es para entrar na lista, sair da lista e usar a lista em
http://www.mat.puc-rio.br/~obmlistas/obm-l.html
=







This message was sent using IMP, the Internet Messaging Program.



--
Esta mensagem foi verificada pelo sistema de antivírus e
acredita-se estar livre de perigo.


=
Instruções para entrar na lista, sair da lista e usar a lista em
http://www.mat.puc-rio.br/~obmlistas/obm-l.html
=


Re: [obm-l] Re: [obm-l] Análise

2015-09-11 Por tôpico gugu
   Desculpem, mas o resultado é falso. Se f(x,n)=sen(nx)/n (digamos  
para x em (1,+infinito)), então quando n tende a infinito, f(x,n)  
tende a 0. Mas df/dx(x,n)=n.cos(nx)/n=cos(nx), que não tende a 0  
quando n tende a infinito. Por exemplo, se x=2.pi, cos(nx)=1 para todo  
n natural.

   Abraços,
 Gugu

Quoting Marcelo Salhab Brogliato <msbro...@gmail.com>:


Oi, Israel,

Acho que a melhor representação seria f(x, n) e g(x, n).

Assim, sua pergunta seria:
Seja h(x, n) = f(x, n) - g(x, n). Prove que, se lim{n->inf} h(x, n) = 0,
então lim{n->inf} dh/dx(x, n) = 0.

Pela hipótese, sabemos que para todo eps > 0 existe M tal que para todo n >
M, |h(x, n)| < eps.

Para eps1*|a| > 0, existe M1 tal que n > M1 implica em |h(x+a, n)| <
eps1*|a|.
Para eps2*|a| > 0, existe M2 tal que n > M2 implica em |h(x, n)| < eps2*|a|.

Portanto, para n > max(M1, M2), temos:
|h(x+a, n) - h(x, n)| <= |h(x+a, n)| + |h(x, n)| < |a|*(eps1 + eps2)
|h(x+a, n) - h(x, n)| < a*(eps1 + eps2)
|h(x+a, n) - h(x, n)| / |a| < eps1 + eps2
|[h(x+a, n) - h(x, n)] / a| < eps1 + eps2

Sabemos que dh/dx = lim{a->0} [h(x+a, n) - h(x, n)] / a.
Fazendo a -> 0, temos |dh/dx(x, a)| < eps1 + eps2, para n > max(M1, M2).

Portanto, lim{n->inf} dh/dx(x, n) = 0.

Abraços,
Salhab

2015-09-11 21:54 GMT-03:00 Israel Meireles Chrisostomo <
israelmchrisost...@gmail.com>:


Seja f(x) uma função na variável  x, mas que contenha n na sua "fórmula"
ou na sua expressão.Vou usar a notação aqui de lim para limite de n
tendendo ao infinito,
se lim f(x) =g(x) então vale que lim f '(x)=lim g'(x)?Como posso provar
isso?Eu usei isto em uma demonstração, mas não sei se está correto...Alguém
poderia me ajudar?

--
Esta mensagem foi verificada pelo sistema de antivírus e
acredita-se estar livre de perigo.


--
Esta mensagem foi verificada pelo sistema de antiv�rus e
 acredita-se estar livre de perigo.







This message was sent using IMP, the Internet Messaging Program.



--
Esta mensagem foi verificada pelo sistema de antiv�rus e
acredita-se estar livre de perigo.


=
Instru��es para entrar na lista, sair da lista e usar a lista em
http://www.mat.puc-rio.br/~obmlistas/obm-l.html
=


Re: [obm-l] Problema

2015-07-09 Por tôpico gugu

   Caro Benedito,
   Encaminho abaixo a solução do Renan Finder, que é ex-olímpico e  
aluno do IMPA, e mostra que A tem estratégia para ganhar:


Chamamos de classe n o conjunto dos números remanescentes que são  
congruentes a n

módulo 5. O jogador A vence se tentar minimizar a quantidade
 |#(classe 1) - #(classe 4)| + |#(classe 2) - #(classe 3)|, que  
chamamos de desequilíbrio.


Prova: inicialmente o desequilíbrio é igual a 2. Após a primeira  
jogada de A, o
desequilíbrio será igual a 1. O jogador B pode, por algumas rodadas,  
aumentar o
desequilíbrio, mas no turno de A ele sempre voltará a ser igual a 1.  
Em alguma jogada, B apagará um número da classe 5 ou tornará o  
desequilíbrio nulo. No primeiro caso, A tornará o desequilíbrio nulo;  
no segundo, restarão quatro números na classe 5 e A poderá apagar um  
deles. A partir daí, A poderá sempre manter o desequilíbrio nulo, logo  
vencerá.


   Abraços,
 Gugu

Quoting bened...@ufrnet.br:


Problema
Dois jogadores, A e B, disputam um jogo, em que jogam   
alternadamente. O jogador A começa. Uma jogada consiste em apagar um  
 dos números inteiros do conjunto {1, 2, 3,..., 27} até que reste   
somente dois números. Se a soma desses dois últimos números for   
divisível por 5, o jogador A vence, caso contrário, vence o jogador B.
Se cada jogador faz suas melhores jogadas, quem vence: A ou B? Qual   
é a estratégia para vencer?


--
Esta mensagem foi verificada pelo sistema de antiv�rus e
 acredita-se estar livre de perigo.







This message was sent using IMP, the Internet Messaging Program.



--
Esta mensagem foi verificada pelo sistema de antiv�rus e
acredita-se estar livre de perigo.


=
Instru��es para entrar na lista, sair da lista e usar a lista em
http://www.mat.puc-rio.br/~obmlistas/obm-l.html
=


Re: [obm-l] Desigualdade

2015-05-04 Por tôpico gugu

C(n,k+1)=n(n-1)...(n-k)/(k+1)!=n^(k+1)/(k+1)!.
Quoting Israel Meireles Chrisostomo israelmchrisost...@gmail.com:


Alguém tem uma demonstração da desigualdade C(n,k+1) = n^(k+1) / (k+1)! de
preferência que não envolva indução hehehe

--
Esta mensagem foi verificada pelo sistema de antiv�rus e
 acredita-se estar livre de perigo.







This message was sent using IMP, the Internet Messaging Program.



--
Esta mensagem foi verificada pelo sistema de antiv�rus e
acredita-se estar livre de perigo.


=
Instru��es para entrar na lista, sair da lista e usar a lista em
http://www.mat.puc-rio.br/~obmlistas/obm-l.html
=


Re: [obm-l] Exponencial e polinômios

2015-05-03 Por tôpico gugu

   Caro Carlos Gomes,
   Se f(x) é derivável então, pelo teorema do valor médio, entre duas  
raízes de f(x) sempre há uma raiz de f'(x). Assim, se f(x) tem pelo  
menos k raízes então f'(x) tem pelo menos k-1 raízes. Temos que  
f(x)=e^x-p(x) é infinitas vezes derivável. Se p(x) for um polinômio de  
grau n, se derivarmos p(x) n+1 vezes dá 0, e logo se derivarmos f(x)  
n+1 vezes dá e^x, que não tem raiz nenhuma. Portanto, f(x) tem no  
máximo n+1 raízes.

   Abraços,
 Gugu

Quoting Carlos Gomes cgomes...@gmail.com:


Olá amigos,

Será que alguém pode me ajudar com essa?

Mostre que para qualquer polinômio com coeficientes reais p(x)  a   
equação e^x=p(x) tem sempre uma quantidade finita de raízes   
(evidentemente pode não ter raízes, caso em que a quantidade de   
raízes é 0 e portanto essa quantidade é finita inclusive nesse caso).


Abraço, Cgomes.

--
Esta mensagem foi verificada pelo sistema de antivírus e
acredita-se estar livre de perigo.

=
Instruções para entrar na lista, sair da lista e usar a lista em
http://www.mat.puc-rio.br/~obmlistas/obm-l.html
=





This message was sent using IMP, the Internet Messaging Program.



--
Esta mensagem foi verificada pelo sistema de antiv�rus e
acredita-se estar livre de perigo.


=
Instru��es para entrar na lista, sair da lista e usar a lista em
http://www.mat.puc-rio.br/~obmlistas/obm-l.html
=


Re: [obm-l] Irracionalidade de pi

2015-05-03 Por tôpico gugu

   Caro Israel,
   Não entendo sua objeção ao argumento do Ralph, que está correto.  
Isso nem seria necessário, mas note que se a_n = Raiz(2)/n para todo  
inteiro positivo n, como a_{n+1}=Raiz(2)/(n+1), temos que  
a_{n+1}=(n/(n+1)).a_n para todo inteiro positivo n. Isso é uma  
recorrência que relaciona (como você queria) cada termo com o anterior  
e que dá uma prova por indução de que a_n é irracional para todo n:  
a_1=Raiz(2) é irracional; se, por hipótese de indução, a_n é  
irracional, então, como n/(n+1) é um racional não-nulo,  
a_{n+1}=(n/(n+1)).a_n também é irracional, c.q.d..

   Mas, no entanto, lim a_n=0 é racional...
   Abraços,
 Gugu

Quoting Israel Meireles Chrisostomo israelmchrisost...@gmail.com:


É mais, no exemplo que vc citou é diferente, pq o fato do termo anterior
ser irracional  não implica uma igualdade entre o termo anterior e o
próximo, isto é, seu contra-exemplo não pode ser aplicado.Minha dúvida é
mais específica, pois a sequência que eu coloquei é uma igualdade que
implica que cada cotangente subsequente é irracional pela igualdade
estabelecida com a cotangente anterior, então isto deve implicar que a
próxima cotangente é irracional.No seu caso, não há uma relação recursiva
que te permita escrever a raiz de 2 sobre n com uma igualdade entre o termo
anterior e o próximo.E sobre o limite de um número irracional ser um número
racional, se vc pegar este seu contra-exemplo não se pode dizer que a
função zeta para valores pares é transcendente, pois esta implicação é
feita através de limites, pois se conclui que a função zeta é transcendente
pq pi é transcendente,isto é  pq zeta é igual a pi^2k multiplicado por um
racional,  o que só pode ser provado por meio de limites, e então, o que vc
me diz? Logo, acredito que esse seu argumento perde o efeito.

Em 2 de maio de 2015 17:54, Ralph Teixeira ralp...@gmail.com escreveu:


Nao funciona... Voce pode ter uma infinidade de numeros irracionais, cujo
limite eh RACIONAL. Pense por exemplo na sequencia

a_n = Raiz(2)/n

Todos esses a_n sao irracionais, mas o limite da sequencia eh 0, um
racional.

Ou seja, como voce suspetaiva, soh porque alguma propriedade vale para
todo n natural, nao significa que ela valha quando n-+Inf.

Abraco, Ralph.

2015-05-02 16:58 GMT-03:00 Israel Meireles Chrisostomo 
israelmchrisost...@gmail.com:


 Olá gente, gostaria de saber se o meu raciocínio para demonstrar a
irracionalidade de pi está correto, a demonstração está no link:


https://docs.google.com/viewer?a=vpid=sitessrcid=ZGVmYXVsdGRvbWFpbnxpc3JhZWxtY2hyaXNvc3RvbW98Z3g6N2I1M2RhZjEwZmZkYmM3Nw

Se alguém puder me ajudar, por favor, me diga algo que eu não saiba, por
exemplo uma justificativa plausível do pq eu não posso aplicar o raciocínio
infinitas vezes da forma como fiz  .A minha dúvida é bem simples, pois se
 eu tivesse partido do princípio que a cotangente ao invés de irracional é
algébrica, pois é raiz do polinômio contido na demonstração, então, eu
poderia ter chegado erroneamente a conclusão de que pi é algébrico plea
igualdade, o que é falso, como contornar isso?A demonstração ainda sim está
correta?

--
Esta mensagem foi verificada pelo sistema de antivírus e
acredita-se estar livre de perigo.




--
Esta mensagem foi verificada pelo sistema de antivírus e
acredita-se estar livre de perigo.


--
Esta mensagem foi verificada pelo sistema de antiv�rus e
 acredita-se estar livre de perigo.







This message was sent using IMP, the Internet Messaging Program.



--
Esta mensagem foi verificada pelo sistema de antiv�rus e
acredita-se estar livre de perigo.


=
Instru��es para entrar na lista, sair da lista e usar a lista em
http://www.mat.puc-rio.br/~obmlistas/obm-l.html
=


Re: [obm-l] Re: [obm-l] Re: [obm-l] Re: [obm-l] Re: [obm-l] Soluções inteiras

2015-04-20 Por tôpico gugu

   Caros,
   Suponhamos que b não é 0 (se for a também tem que ser). Dado p  
primo, se p^k é a maior potência de p que divide b, e p^j é a maior  
potência de p que divide a, como a^13=b^2001-b^90, p^(90k) é a maior  
potência de p que divide a^13, ou seja, p^(90k)=p^(13j), donde  
90k=13j, e logo k=13r, j=90r para algum inteiro positivo r. Assim, se  
m é o produto dos primos que dividem a mas não dividem b (que em  
princípio poderiam existir) pelo sinal de a (que poderia ser  
negativo), devemos ter
b^2001-b^90=a^13=b^90.m^13, donde b^1911-1=m^13, ou seja,  
(b^637)^3-m^3=1. Como os únicos jeitos de a diferença de dois cubos de  
inteiros ser igual a 1 são 1^3-0^3 e 0^3-(-1)^3, devemos ter b^637=1 e  
m=0 (donde b=1 e a=0) ou b^637=0 (donde b=0 e a=0).

   Abraços,
 Gugu

Quoting Pacini Bores pacini.bo...@globo.com:


Ok!  Pedro, obrigado pela observação do expoente de p  em |b| não ser
necessariamente igual a 1. A sua conclusão foi estratégica.

Abraços

Pacini

Em 20 de abril de 2015 10:23, Pedro José petroc...@gmail.com escreveu:


Douglas,

desculpe-me, só havia visto a nota do Pacini a equação original é
a^13+b^90=b^2001 então (0,0) também é solução.

Saudações,
PJMS

Em 20 de abril de 2015 10:12, Pedro José petroc...@gmail.com escreveu:


Bom dia!

Se há um fator p primo na fatoração de |b| então p é fator primo de |a|,
está correto.
Porém, o fator em b não é necessariamente 1, pode ser y e aí há solução
13 x - 90 y = 0.
Só que |a|^13 = b^90 == |b^1911-1| = 1 o que é absurdo.
então só há solução para a=0 == b=1.

Douglas,

(0,0) não é solução embora possa parecer contraditório 0 divide 0, porém
não existe divisão por zero.

a divide b se existe k  Ɛ Z | b = ka.

Porém, x/y == y ǂ 0

Saudações,
PJMS


Em 19 de abril de 2015 19:02, Douglas Oliveira de Lima 
profdouglaso.del...@gmail.com escreveu:


Mas (a,b)=(0,0), ou (a,b)=(0,1) são soluções, então neste caso seriam
somente essas.

Em 18 de abril de 2015 20:28, Pacini Bores pacini.bo...@globo.com
escreveu:


Olá, por favor me corrijam se estou pensando errado.

(a^13)/(b^90) = (b^1911 - 1). Seja p um fator primo de |b|, então p é
um fator primo de |a|, ok ?
Logo o fator primo  p deve aparecer com expoente tal que  o lado
esquerdo da igualdade acima não tenha fator primo p, já que o   
lado direito

não é  divisível por p.

Seja então  x  o expoente de p em  |a|, donde teremos do lado
esquerdo o valor 13x-90 como expoente de p, o que é estranho pois esse
expoente é maior do que ou igual a 1. Daí não poderemos ter   
soluções, pois

p não divide o lado direito da igualdade acima.

Abraços

Pacini




Em 18 de abril de 2015 18:56, Douglas Oliveira de Lima 
profdouglaso.del...@gmail.com escreveu:


Olá caros amigos gostaria de uma ajuda nesta questão, quais são as
soluções inteiras da equação a^13+b^90=b^2001.

Agradeço Desde já.
Douglas Oliveira

--
Esta mensagem foi verificada pelo sistema de antivírus e
acredita-se estar livre de perigo.




--
Esta mensagem foi verificada pelo sistema de antivírus e
acredita-se estar livre de perigo.




--
Esta mensagem foi verificada pelo sistema de antivírus e
acredita-se estar livre de perigo.






--
Esta mensagem foi verificada pelo sistema de antivírus e
acredita-se estar livre de perigo.



--
Esta mensagem foi verificada pelo sistema de antiv�rus e
 acredita-se estar livre de perigo.







This message was sent using IMP, the Internet Messaging Program.



--
Esta mensagem foi verificada pelo sistema de antiv�rus e
acredita-se estar livre de perigo.


=
Instru��es para entrar na lista, sair da lista e usar a lista em
http://www.mat.puc-rio.br/~obmlistas/obm-l.html
=


Re: [obm-l] Dúvida sobre Álgebra

2015-04-13 Por tôpico gugu

   Saudações.
   A sua afirmação é equivalente a dizer que 3, 5, 17, 257 e 65537  
são os únicos primos de Fermat (o que está em aberto, e muitos  
matemáticos consideram provável). Se F_n=2^{2^n}+1, F_n-2=2^{2^n}-1 é  
o produto dos F_k de k=0 até n-1 (por exemplo, 255=3*5*17), o que pode  
ser facilmente provado por indução (pois
F_n.(F_n-2)=F_{n+1}-2). Sabemos que F_5=4294967297 não é primo - é  
múltiplo, por exemplo, de 641 (e portanto não é possível construir com  
régua e compasso um polígono regular com F_5 lados). Como eu mencionei  
acima, não se conhece nenhum primo de Fermat F_n maior que F_4=65537,  
mas, se existir, F_n-2 seria múltiplo de F_5, e logo não seria  
possível dividir com régua e compasso uma circunferência em F_n-2  
arcos congruentes (senão seria possível construir com régua e compasso  
um polígono regular com F_5 lados).

   Abraços,
 Gugu

Quoting Listeiro 037 listeiro_...@yahoo.com.br:




Saudações.

Tenho a dúvida sobre como se pode demonstrar (se for realmente verdade)
que se 'p' é primo e divide uma circunferência com instrumentos
euclidianos, então p-1 e p-2 também a divide. Ou seja, se existirem
infinitos pp então existem infinitas tríades de consecutivos. Na
verdade p tem que ser um número de Fermat. E p-2 é um produto de
números de Fermat, se estiver correto.

Grato a quem puder me orientar.

--
Esta mensagem foi verificada pelo sistema de antiv�rus e
 acredita-se estar livre de perigo.


=
Instru��es para entrar na lista, sair da lista e usar a lista em
http://www.mat.puc-rio.br/~obmlistas/obm-l.html
=







This message was sent using IMP, the Internet Messaging Program.



--
Esta mensagem foi verificada pelo sistema de antiv�rus e
acredita-se estar livre de perigo.


=
Instru��es para entrar na lista, sair da lista e usar a lista em
http://www.mat.puc-rio.br/~obmlistas/obm-l.html
=


Re: Re: [obm-l] É soma de dois quadrados

2015-03-11 Por tôpico gugu

   Caros,
   Não é verdade a afirmação do Listeiro 037 que diz que ..números  
da forma 4n+1 podem ser escritos como soma de dois quadrados. Por  
exemplo, 21=4*5+1 não pode ser escrito como soma de dois quadrados. A  
condição necessária e suficiente para que um natural possa ser escrito  
como soma de dois quadrados (de naturais, podendo um deles ser 0) é  
que, em sua fatoração como produto de potências de primos distintos,  
os expoentes dos primos da forma 4k+3 sejam sempre pares (e 21=3*7 não  
satisfaz essa condição.
   Por outro lado, em relação ao problema original, se  
(n+1)^3-n^3=3n^2+3n+1=k^2, temos (6n+3)^2+3=36n^2+36n+12=12k^2, ou  
seja, 12k^2-3=3(4k^2-1)=3(2k+1)(2k-1) é um quadrado perfeito ímpar.  
Como 2k+1 e 2k-1 são primos entre si, existem u e v ímpares tais que  
(2(2k+1)=u^2 e 2k-1=v^2) ou (2k+1=u^2 e 3(2k-1)=v^2). Devemos ter k  
ímpar, senão 12k^2-3 seria um número da forma 8m-3, o qual não poderia  
ser quadrado perfeito, absurdo. Daí, 2k+1 é um número da forma 4n+3, e  
logo não pode ser quadrado perfeito. Devemos então ter,  
necessariamente, 2(2k+1)=u^2 e 2k-1=v^2. Assim, v é ímpar, digamos  
v=2w+1, e 2k-1=v^2=(2w+1)^2=4w^2+4w+1, donde k=2w^2+2w+1=(w+1)^2+w^2.  
Assim, (n+1)^3-n^3=3n^2+3n+1=k^2=((w+1)^2+w^2)^2=(2w+1)^2+(2w(w+1))^2.

   Abraços,
 Gugu

Quoting Listeiro 037 listeiro_...@yahoo.com.br:



Obrigado. Agora está mais fácil enxergar.

Em Tue, 10 Mar 2015 13:55:38 -0300
Pedro José petroc...@gmail.com escreveu:


Boa tarde!

49 = 4*12 + 1 e como escrever 49 como a soma de dois quadrados de
naturais, excetuando-se o zero?
Embora não mencionado no enunciado, deveria ser estritamente naturais
o universo; pois a^2 = a^2 + 0^2.

 Ai fica atendido sempre.

Creio que se deva enquadrar o quadrado, sempre que acontecer, em uma
das tríades pitagóricas (2st; s^2-t^2; s^2+t^2) primitivas ou
qualquer das tríades pitagóricas (2nst; n*(s^2-t^2) ; n*(s^2+t^2));
s,t, n Ɛ  |N* e n1. |N* = {1,2,3,4...}

Ou que haja um conjunto finito com poucos elementos, e portanto fácil
de se analisar as tríades.

É mais complicado que a solução proposta.

Saudações,
PJMS


Em 10 de março de 2015 08:54, Listeiro 037 listeiro_...@yahoo.com.br
escreveu:


 Quadrados, por natureza, são da forma 4n ou da forma 4n+1.
 Como são números consecutivos, essa diferença é ímpar, portanto da
 forma 4n+1.
 E como números da forma 4n+1 podem ser escritos como soma de dois
 quadrados, a afirmativa é verdadeira.

 Agora, por gentileza ... é simples assim mesmo ou eu caí em algum
 resultado vago ... por favor ... corrijam-me. Essa argumentação é
 suficiente?

 Em Sun, 8 Mar 2015 14:23:12 +
 marcone augusto araújo borges marconeborge...@hotmail.com
 escreveu:

  Mostre que a diferença dos cubos de dois naturais consecutivos
  quando é quadrado é soma de dois quadradosExemplo: 8^3 - 7^3 =
  (2^2 + 3^2)^2 3x^3 + 3x + 1 = t^2 Delta = 12t^2 - 3 = 3(4t^2 -
  1)4t^2 - 1 = 3k^2(2t)^2 - 3k^2 = 1Empaquei.
 

 --
 Esta mensagem foi verificada pelo sistema de antivírus e
  acredita-se estar livre de perigo.


 =
 Instru�ões para entrar na lista, sair da lista e usar a lista em
 http://www.mat.puc-rio.br/~obmlistas/obm-l.html
 =




--
Esta mensagem foi verificada pelo sistema de antiv�rus e
 acredita-se estar livre de perigo.


=
Instru��es para entrar na lista, sair da lista e usar a lista em
http://www.mat.puc-rio.br/~obmlistas/obm-l.html
=







This message was sent using IMP, the Internet Messaging Program.



--
Esta mensagem foi verificada pelo sistema de antiv�rus e
acredita-se estar livre de perigo.


=
Instru��es para entrar na lista, sair da lista e usar a lista em
http://www.mat.puc-rio.br/~obmlistas/obm-l.html
=


Re: [obm-l] Convergência/divergência de uma sequência e de uma série

2014-11-07 Por tôpico gugu

   Cara Amanda,
   Como as respostas que chegaram às suas perguntas foram  
incompletas, resolvi escrever soluções, ainda que com uma certa demora  
(pelo que peço desculpas). Vamos lá:

a) Seja f:[1, oo) decrescente e limitada e seja (a_n) dada por
 a_n = Soma(k = 1, n) f(k) - Int [1, n] f(x) dx, n = 1, 2,3 .
 Mostre que (a_n) converge (mesmo que a série e a integral divirjam.  
Em caso de
convergência de ambas, o resultado é imediato. Aliás, pelo teste da  
integral, ou ambas convergem ou ambas divergem)


R.: Podemos escrever a_n=Soma(k = 1, n) b_k, onde b_1=f(1) e  
b_k=f(k)-Int [k-1, k] f(x) dx= Int [k-1, k] (f(k)-f(x)) dx para k1.  
Como f é decrescente, temos b_k0, pois f(k)-f(x)0 para k-1=xk.  
Assim, a sequência a_n é decrescente. Por outro lado, como  
f(k)-f(x)=f(k)-f(k-1) para k-1=x=k, temos b_k=f(k)-f(k-1) para  
k1, donde a_n=Soma(k = 1, n) b_k=f(1)+Soma(k = 1,  
n)(f(k)-f(k-1)=f(n). Como f é limitada, (a_n) é decrescente e  
limitada, e logo converge.


 b) Seja (a_n) uma sequência de reais positivos e (s_n) a sequência  
de suas somas
parciais. Estude a convergência/divergência de Soma (a_n)/(s_n) para  
os seguintes casos:


 b.1) a_n = 1/n^2, n = 1, 2, 3

 b.2) a_n = 1/(p_n), sendo p_n o n-ésimo primo.

R.: Vamos provar que, para qualquer sequência (a_n) de reais  
positivos, a série de
(a_n)/(s_n) converge se, e somente se, a série de a_n converge. De  
fato, se a série de a_n converge, como s_n=a_1 para todo n, a soma de  
a_n/s_n é menor ou igual à soma de a_n/a_1, que é limitada, e portanto  
converge. Por outro lado, se a série de a_n diverge, para todo inteiro  
positivo m existe nm tal que s_n2.s_m, e logo a soma de k=m+1 até n  
de a_k/s_k é maior ou igual à soma de k=m+1 até n de a_k/s_n, que é  
igual a (s_n-s_m)/s_n=1-s_m/s_n1/2. Assim, para todo m (por maior que  
seja), a soma de alguns termos da série a_k/s_k com km é maior que  
1/2, o que implica que a série de a_k/s_k diverge. Vamos agora tratar  
dos subitens:

b.1) Como a série de 1/n^2 converge, a série de (a_n)/(s_n) também converge.
b.2) Vamos mostrar que a série dos inversos dos primos diverge, e  
portanto a série de (a_n)/(s_n) também diverge. Para isso vou mostrar  
um argumento bacana do Erdös:
Se a série de 1/p_n convergisse, existiria N natural tal que a série  
dos 1/p_n com nN seria menor que 1/2, Seja A o conjunto dos naturais  
que têm algum fator primo p_n com nN, e B o complementar de A nos  
naturais. Dado m um inteiro positivo, o número de inteiros positivos  
entre 1 e m que pertencem a A é no máximo a soma de m/p_n para nN,  
que é menor que m/2. Por outro lado, um elemento de B menor ou igual a  
m se escreve como p_1^a_1.p_2^a_2.p_N^a_N (não pode ter nenhum  
fator primo maior que p_N), e os expoentes a_j são no máximo log m/log  
2. Assim, há no máximo (1+log m/log 2)^N possibilidades para esses  
elementos. Como há m inteiros positivos entre 1 e m, devemos ter  
m/2+(1+log m/log 2)^Nm, donde (1+log m/log 2)^Nm/2 para todo inteiro  
ositivo m, o que é um absurdo, pois o limite de (1+log m/log 2)^N/m  
quando m tende a infinito é 0.

   Abraços,
 Gugu

Quoting Amanda Merryl sc...@hotmail.com:

Boa noite. Estou com alguma dificuldade nisto. Agradeço se puderem   
ajudar em um deles.


a) Seja f:[1, oo) decrescente e limitada e seja (a_n) dada por

a_n = Soma(k = 1, n) f(k) - Int [1, n] f(x) dx, n = 1, 2,3 .

Mostre que (a_n) converge (mesmo que a série e a integral divirjam.   
Em caso de convergência de ambas, o resultado é imediato. Aliás,   
pelo teste da integral, ou ambas convergem ou ambas divergem)


b) Seja (a_n) uma sequência de reais positivos e (s_n) a sequência   
de suas somas parciais. Estude a convergência/divergência de Soma   
(a_n)/(s_n) para os seguintes casos:


b.1) a_n = 1/n^2, n = 1, 2, 3

b.2) a_n = 1/(p_n), sendo p_n o ngésimo primo.

Muito obrigada

Amanda.



--
Esta mensagem foi verificada pelo sistema de antiv�rus e
 acredita-se estar livre de perigo.


=
Instru��es para entrar na lista, sair da lista e usar a lista em
http://www.mat.puc-rio.br/~obmlistas/obm-l.html
=







This message was sent using IMP, the Internet Messaging Program.



--
Esta mensagem foi verificada pelo sistema de antiv�rus e
acredita-se estar livre de perigo.


=
Instru��es para entrar na lista, sair da lista e usar a lista em
http://www.mat.puc-rio.br/~obmlistas/obm-l.html
=


Re: [obm-l] Problema de Álgebra

2014-10-29 Por tôpico gugu

   Cara Mariana,
   Acho que há algum problema com o enunciado. Seja n=122=2.61. Se  
escrevemos dois algarismos após o algarismo das unidades de n obtemos  
um número entre 12200 e 12299. Como 110^2=1210012200 e  
111^2=1232112299, nenhum desses números é um quadrado perfeito.

   Abraços,
 Gugu

Quoting Mariana Groff bigolingroff.mari...@gmail.com:


Boa tarde,
Não consigo resolver o problema a seguir, alguém poderia me ajudar?

O inteiro n é o produto de dois inteiros positivos. Prove que

(a) é possível escrever dois algarismos após os algarismos das unidades
deste número de modo que o inteiro resultante seja um quadrado perfeito.

(b) se n12 então só existe uma maneia de escolher estes algarismos.

Obrigada!

--
Esta mensagem foi verificada pelo sistema de antiv�rus e
 acredita-se estar livre de perigo.







This message was sent using IMP, the Internet Messaging Program.



--
Esta mensagem foi verificada pelo sistema de antiv�rus e
acredita-se estar livre de perigo.


=
Instru��es para entrar na lista, sair da lista e usar a lista em
http://www.mat.puc-rio.br/~obmlistas/obm-l.html
=


Re: [obm-l] Equacao funcional.

2014-08-26 Por tôpico gugu

   Caro Douglas,
   Fazendo y=f(x): f(x^2+f(x))+f(0)=2f(f(x))+2f(x)^2.
   Fazendo y=-x^2: f(0)+f(f(x)+x^2)=2f(f(x))+2x^4.
   Comparando, temos f(x)^2=x^4, donde, para todo x, f(x)=x^2 ou  
f(x)=-x^2. Em particular, f(0)=0. Fazendo então x=0 temos  
f(y)+f(-y)=2y^2, mas f(y) e f(-y) pertencem a {y^2, -y^2}, donde  
necessariamente f(y)=f(-y)=y^2. Assim, f(x)=x^2 para todo x real.

   Abraços,
 Gugu

Quoting Douglas Oliveira de Lima profdouglaso.del...@gmail.com:


Caos amigos preciso de uma ajuda na seguinte questão, desde ja agradeço!!

Problema:  Se f(x^2+y)+f(f(x)-y)=2f(f(x))+2y^2 para todos x,t pertencentes
aos reais, determinar todas as funções f:R-R.

Douglas Oliveira.

--
Esta mensagem foi verificada pelo sistema de antiv�rus e
 acredita-se estar livre de perigo.







This message was sent using IMP, the Internet Messaging Program.



--
Esta mensagem foi verificada pelo sistema de antiv�rus e
acredita-se estar livre de perigo.


=
Instru��es para entrar na lista, sair da lista e usar a lista em
http://www.mat.puc-rio.br/~obmlistas/obm-l.html
=


[obm-l] Alguns problemas de Teoria dos Números (comunicados pelo Claudio Buffara)

2012-01-21 Por tôpico gugu

   Caros amigos e colegas,
   Meu amigo Claudio Buffara me enviou recentemente alguns problemas  
de Teoria dos Números bastante simpáticos, que eu gostaria de  
compartilhar com vocês:


1) Prove que, dado qualquer polinômio f(x) em Z[x], existe um natural N
(dependente de f(x)) tal que se p é um primo maior do que N, então:
x*f(x) + p   e   x*f(x) - p são irredutíveis.

2) Prove que a funcão f: N - N  (N = conj. dos naturais) dada por:
f(n) = n*Phi(n)   (Phi = função de Euler) é injetiva.

2') Provar que, se m e n são distintos e livres de quadrados, então
m*sigma(m)  n*sigma(n).

3) Provar que a sequência 1, 3, 2, 6, 8, 4, 11, 5, ...  (o n-ésimo termo é
o menor natural que ainda não apareceu na sequência e é tal que a soma dos n
primeiros termos é divisível por n) contém todos os naturais.

   Se vocês quiserem que eu mande soluções por favor escrevam para  
g...@impa.br (eu tenho olhado muito irregularmente a lista, por falta  
de tempo).


   Abraços,
 Carlos Gustavo (Gugu)


This message was sent using IMP, the Internet Messaging Program.



=
Instruções para entrar na lista, sair da lista e usar a lista em
http://www.mat.puc-rio.br/~obmlistas/obm-l.html
=


[obm-l] Fwd: [Prime] Two new Mersenne primes announced

2008-09-16 Por tôpico gugu



- Forwarded message from [EMAIL PROTECTED] -
Date: Tue, 16 Sep 2008 08:38:35 -0400
From: George Woltman [EMAIL PROTECTED]
Reply-To: The Great Internet Mersenne Prime Search list [EMAIL PROTECTED]
 Subject: [Prime] Two new Mersenne primes announced
  To: The Great Internet Mersenne Prime Search list [EMAIL PROTECTED]

The two new Mersenne primes are M43112609 and M37156667.  You can read
all about their discovery at http://mersenne.org

Congratulations to all GIMPS members for their part in making this happen!!
___
Prime mailing list
[EMAIL PROTECTED]
http://hogranch.com/mailman/listinfo/prime


- End forwarded message -



This message was sent using IMP, the Internet Messaging Program.


=
Instruções para entrar na lista, sair da lista e usar a lista em
http://www.mat.puc-rio.br/~obmlistas/obm-l.html
=


Re: RES: [obm-l] Cj. Cantor

2006-07-06 Por tôpico gugu

   Caro Artur,
   Vou tentar explicar algumas dessas coisas:
Quoting Artur Costa Steiner [EMAIL PROTECTED]:


Com relacao a este assunto, eu gostaria que alguem esclarecesse algumas
duvidas, fiquei um tanto confuso:

A dimensao de Hausdorff eh baseada na medida (ou medida exterior, nao estou
certo) de Hausdorff, OK? Se A eh um conjunto de R^n  eh H_d(A)eh a sua
medida d-dimensional de Hausdorff, entao a dimensao de Hausdorff D(A) eh
definida por infimo{d =0 | H_d(A) 0}, certo? A medida de Hausdorff eh uma
extensao da medida de Lebesgue, soh que em vez da soma dos volumes (volume,
aqui, no sentido geral) dos conjuntos da cobertura, tomam-se as potências d
deste volumes,


Mais precisamente toma-se potências d dos diâmetros dos conjuntos da 
cobertura.



limitando o diametro dos conjuntos da cobertura em r0 e ,
depois , tomando-se o infimo em r=0 desta somas de potencias d dos volumes.


E fazendo r tender a 0, ou seja, tomando o liminf dessas somas quando r 
tende a 0.



Assim, para d=1 as medidas de Hausdorff e de Lebesgue se confundem, certo?

Certo.

Se H_d(A)  oo, entao H_p(A) = 0 para p d e H_p(A) = oo para 0= p d. Eh
isso mesmo?


Sim.


O Gugu dise  que existe um conjunto de cantor K com dimensao de Hausdorff
nula e tal que K - K seja um intervalo. Isto implica que, em R^n, eh
possivel que um conjunto A tenha medida de Lebesgue nula e que, ainda assim,
A- A contenha uma bola, certo?


Sim. Mas isso já segue de tomar A igual ao conjunto de Cantor usual 
(que tem dimensão de Hausdorff positiva mas medida de Lebesgue nula).




Tambem eh verdade em R^n que, se A tem medida positiva (compacto ou nao),
entao A - A contem uma bola centrada na origem e A + A contem uma bola em
algum lugar, certo?


Sim.



Existe o classico conjunto de Cantor, obtido removendo-se sucessivamente
tercos de intervalos de [0, 1], o qual eh compacto e tem medida nula, logo
interior vazio. Mas falou-se em conjuntos de Cantor, logo existem outros
que ateh podem conter intervalos. Como sao construidas estas generalizacoes
do conjunto basico de Cantor?


Conjuntos de Cantor são conjuntos homeomorfos ao conjunto de Cantor 
usual, e portanto não contêm intervalos. Os conjuntos de Cantor 
contidos na reta real são  exatamente os conjuntos compactos, sem 
pontos isolados e de interior vazio.

   Abraços,
 Gugu


Obrigado

Artur


-Mensagem original-
De: [EMAIL PROTECTED] [mailto:[EMAIL PROTECTED]
nome de [EMAIL PROTECTED]
Enviada em: terça-feira, 4 de julho de 2006 20:00
Para: Nicolau C. Saldanha
Cc: obm-l@mat.puc-rio.br
Assunto: Re: RES: [obm-l] Cj. Cantor


Oi Nicolau,
Na verdade, estritamente falando, a sua afirmação não é verdadeira:
é possível exibir um conjunto de Cantor A na reta com dimensão de
Hausdorff 1 tal que A-A tem medida nula. O meu trabalho com o Yoccoz,
no qual provamos uma conjectura do Jacob, implica que a maioria
(aberto, denso e de medida total)dos conjuntos de Cantor
dinamicamente definidos por funções expansoras (pelo menos de classe
C^(1+d), com d0; para C^1 é genericamente falso) K com dimensão de
Hausdorff maior que 1/2 é tal que K-K tem interior não-vazio. Por outro
lado, para Conjuntos de Cantor bem-comportados (por exemplo
dinamicamente definidos) K com dimensão de Hausdorff menor que 1/2, K-K
tem medida nula. Isso vale sempre que a capacidade limite (que, para
conjuntos bem-comportados, coincide com a dimensão de Hausdorff) de K é
menor que 1/2. Por outro lado é possível construir um conjunto de
Cantor K na reta com domensão de Hausdorff 0 tal que K-K é um intervalo.
Abraços,
   Gugu

Quoting Nicolau C. Saldanha [EMAIL PROTECTED]:


On Tue, Jul 04, 2006 at 11:28:39AM -0300, Artur Costa Steiner wrote:

Esta conclusao a respeito do conjunto K de Cantor eh exemplo de uma
conclusao interessante. Sabemos que se um conjunto A de R^n tem medida de
Lebesgue positiva, entao A - A contem uma bola centrada na origem. Mas a
reciproca na eh vedadeira. K tem medida nula e, mesmo assim, K - K = [-1,
1], que eh uma bola em R centrada na origem.


Novamente, o Gugu é a autoridade no assunto, mas existe um número a,
0  a  1, tal que se a dimensão de Hausdorff de um subconjunto compacto
A de R é maior do que a então A-A contem uma bola centrada na origem.
Infelizmente eu não sei qual é o menor valor de a para o qual vale
este resultado, nem sei se o nosso exemplo segue deste resultado geral.

Se A tem medida positiva sua dimensão de Hausdorff é 1;
a dimensão de Hausdorff de K (o conjunto de Cantor usual)
é log 2/log 3 ~= 0.63.

[]s, N.






This message was sent using IMP, the Internet Messaging Program.


=
Instruções para entrar na lista, sair da lista e usar a lista em
http://www.mat.puc-rio.br/~nicolau/olimp/obm-l.html

Re: RES: [obm-l] Cj. Cantor

2006-07-04 Por tôpico gugu

   Oi Nicolau,
   Na verdade, estritamente falando, a sua afirmação não é verdadeira: 
é possível exibir um conjunto de Cantor A na reta com dimensão de 
Hausdorff 1 tal que A-A tem medida nula. O meu trabalho com o Yoccoz, 
no qual provamos uma conjectura do Jacob, implica que a maioria 
(aberto, denso e de medida total)dos conjuntos de Cantor 
dinamicamente definidos por funções expansoras (pelo menos de classe 
C^(1+d), com d0; para C^1 é genericamente falso) K com dimensão de 
Hausdorff maior que 1/2 é tal que K-K tem interior não-vazio. Por outro 
lado, para Conjuntos de Cantor bem-comportados (por exemplo 
dinamicamente definidos) K com dimensão de Hausdorff menor que 1/2, K-K 
tem medida nula. Isso vale sempre que a capacidade limite (que, para 
conjuntos bem-comportados, coincide com a dimensão de Hausdorff) de K é 
menor que 1/2. Por outro lado é possível construir um conjunto de 
Cantor K na reta com domensão de Hausdorff 0 tal que K-K é um intervalo.

   Abraços,
  Gugu

Quoting Nicolau C. Saldanha [EMAIL PROTECTED]:


On Tue, Jul 04, 2006 at 11:28:39AM -0300, Artur Costa Steiner wrote:

Esta conclusao a respeito do conjunto K de Cantor eh exemplo de uma
conclusao interessante. Sabemos que se um conjunto A de R^n tem medida de
Lebesgue positiva, entao A - A contem uma bola centrada na origem. Mas a
reciproca na eh vedadeira. K tem medida nula e, mesmo assim, K - K = [-1,
1], que eh uma bola em R centrada na origem.


Novamente, o Gugu é a autoridade no assunto, mas existe um número a,
0  a  1, tal que se a dimensão de Hausdorff de um subconjunto compacto
A de R é maior do que a então A-A contem uma bola centrada na origem.
Infelizmente eu não sei qual é o menor valor de a para o qual vale
este resultado, nem sei se o nosso exemplo segue deste resultado geral.

Se A tem medida positiva sua dimensão de Hausdorff é 1;
a dimensão de Hausdorff de K (o conjunto de Cantor usual)
é log 2/log 3 ~= 0.63.

[]s, N.






This message was sent using IMP, the Internet Messaging Program.


=
Instruções para entrar na lista, sair da lista e usar a lista em
http://www.mat.puc-rio.br/~nicolau/olimp/obm-l.html
=


Re: [obm-l] P.A

2006-03-14 Por tôpico gugu
Quoting Nicolau C. Saldanha [EMAIL PROTECTED]:

 On Tue, Mar 14, 2006 at 01:11:41AM +, Klaus Ferraz wrote:
  Existe uma progressao aritmetica infinita de razao diferente de zero que
 pode
  ser formada apenas por numeros primos ? Prove

 Em outras palavras, o problema pergunta se existem inteiros positivos a e b
 tais que an+b seja primo para todo inteiro positivo n. A resposta é não:
 tome n = b; temos an+b = (a+1)*b.

 Dá para demonstrar de forma não muito diferente que qualquer polinômio de
 coeficientes inteiros P(n) assume valores compostos para infinitos valores
 de n.

 Outros problemas bem mais difíceis são:

 * existem progressões aritméticas arbitrariamente longas formadas por primos?

 Isso foi provado há uns 2 anos por Ben Green e Terence Tao, dois ex-olímpicos,
e de fato não é nada fácil.


 * existem infinitos primos da forma n^2 + 1?

  Já esse, que eu saiba, ainda está em aberto. Una interpretação que eu acho
simpática desse problema é se existem infinitos primos (ou irredutíveis, como
queiram) na PA de inteiros de Gauss n+i=i+n.1, onde n percorre os inteiros.
  Abraços,
Gugu

 []s, N.
 =
 Instruções para entrar na lista, sair da lista e usar a lista em
 http://www.mat.puc-rio.br/~nicolau/olimp/obm-l.html
 =






This message was sent using IMP, the Internet Messaging Program.

=
Instruções para entrar na lista, sair da lista e usar a lista em
http://www.mat.puc-rio.br/~nicolau/olimp/obm-l.html
=


Re: [obm-l] limite

2006-02-14 Por tôpico gugu
   Dá para mostrar que, por exemplo, a_n500 e b_n400 para todo n, por indução:
claramente isso vale para n=1, e, se vale para n, temos
a_(n+1) = 300 + 0,3 b_n  300+0,3.400  500, e
b_(n+1) = 200 + 0,3 a_(n+1)  200+0,3.500  400.
   Abraços,
 Gugu


Citando Marcelo Salhab Brogliato [EMAIL PROTECTED]:

 Olá,
 b_n = 290 + 0,09 b_(n-1)

 mas b1 = 290.. logo:

 b_n = b1 + 0,09 b_(n-1)

 Abrindo para n = 2, n = 3, vc vai ver a seguinte lei de formacao:

 b_n = Sum(i=0 ... n-1, (0,09)^i) b1

 Essa somatorio, eh um somatorio de PG e vale: [ 1 - (0,09)^n ] / 0,81
 Assim: b_n = [ 1 - (0,09)^n ] / 0,81 * b1

 Para confirmar, por inducao:
 n=1 = b1 = 0,81 / 0,81 * b1 = b1 .. ok!
 Suponhamos verdadeiro para n, e vamos mostrar que vale para n+1..

 b_(n+1) = b1 + 0,09 b_n = b1 + 0,09 [ 1 - (0,09)^n ] / 0,81 * b1 = [ 0,81 +
 0,09 - (0,09)^(n+1) ] / 0,81 * b1 = [ 1 - (0,09)^(n+1) ] / 0,81 * b1

 Ok.. demonstrado!

 lim b_n = b1 / 0,81
 que é o resultado que ja tinhamos obtido anteriormente...
 tentei mostrar que b_n converge por outros metodos (sem obter o termo geral)
 ... consegui provar que ele é crescente..
 mas nao consegui provar que é limitado! =/
 se alguem souber um modo mais simples de provar, mandai!

 abraços,
 Salhab

   - Original Message -
   From: Klaus Ferraz
   To: obm-l@mat.puc-rio.br
   Sent: Friday, February 10, 2006 7:32 PM
   Subject: Re: [obm-l] limite


   Ola Marcelo,
legal a solucao. Mas como provo q as series bn e an convergem?

   Marcelo Salhab Brogliato [EMAIL PROTECTED] escreveu:
 a1 = 300
 b1 = 200 + 0,3 a1

 a2 = 300 + 0,3 b1
 b2 = 200 + 0,3 a2

 a_n = 300 + 0,3 b_(n-1)
 b_n = 200 + 0,3 a_n

 substituindo a_n em b_n, temos:
 b_n = 2! 00 + 0,3 [ 300 + 0,3 b_(n-1) ]
 b_n = 200 + 90 + 0,09 b_(n-1)
 b_n = 290 + 0,09 b_(n-1)

 Supondo que b_n converge, temos que lim b_n = lim b_(n-1).. assim:
 lim b_n = 290 + 0,09 lim b_n
 lim b_n = 290 / [1 - 0,09] = 290 / 0,91 = 318,68 = 318 alunos

 lim a_n = 300 + 0,3 lim b_n = 300 + 0,3 * 318,68 = 395,60 = 395 alunos

 faltou provar que as series convergem.. mas nao eh dificil.. olhe:
 se provarmos que b_n converge, então, necessariamente, a_n converge...
 fica co! mo exercicio provar que b_n converge..

 abraços,
 Salhab
   - Original Message -
   From: Klaus Ferraz
   To: obm-l@mat.puc-rio.br
   Sent: Thursday, February 09, 2006 8:55 PM
   Subject: [obm-l] limite


   Uma faculdade recebe todos os anos 300 alunos novos no primeiro
 semestre e 200 alunos novos no segundo semestre. 30% dos alunos sao
 reprovados no primeiro periodo e repetem o periodo no semestre seguinte.
 Sendo an e bn respectivamente os numeros de alunos do primeiro periodo no
 primeiro e segundo semestres do ano n, calcule lim(n--infinito) an e lim
 (n--infinito)bn.

 --
   Yahoo! Search
   Dê uma espiadinha e saiba tudo sobre o Big Brother Brasil.




 --
   Yahoo! doce lar. Faça do Yahoo! sua homepage.





This message was sent using IMP, the Internet Messaging Program.

=
Instruções para entrar na lista, sair da lista e usar a lista em
http://www.mat.puc-rio.br/~nicolau/olimp/obm-l.html
=


Re: [obm-l] TEOREMA FUNDAMENTAL DA ARITMÉTICA

2006-02-06 Por tôpico gugu
Vamos lá:

Citando [EMAIL PROTECTED]:


 Mostrar que:


 a) SE r é raiz da equação 10^r=2 entao r é irracional.

Se r fosse racional, digamos r=p/q com p e q inteiros positivos (note que r é
positivo), teríamos 10^(p/q)=2, donde 10^p=2^q, absurdo, pois 10^p é múltiplo
de 5, mas 2^q não é.


 b) Se n pertence aos naturais é livre de quadrado, isto é, não existe numero
 primo p tal que p^2 divide n, então a raiz de n é irracional.

Isto é falso se n=1, que é livre de quadrados e tem raiz inteira. Se n1 é
verdade, e segue do item c) abaixo: se a raiz de n fosse racional, seria
inteira; se k é a raiz de n, n=k^2, e, como n, temos k1, donde existe p primo
que divide k, e logo p^2 divide k^2=n, e logo n não é livre de quadrados.


 c)Se raiz de n é racional, então raiz de n é inteiro.

Imagino que aqui n denote um número natural. Se raiz de n=p/q onde p e q são
inteiros positivos tais que mdc(p,q)=1, temos n=p^2/q^2. De mdc(p,q)=1 segue
que mdc(p^2,q^2)=1 (de fato, os fatores primos de p^2 são os mesmos fatores
primos de p, e os fatores primos de q^2 são os mesmos de q). Assim, como
p^2/q^2=n é inteiro, temos necessariamente q=1 (pois p^2=q^2.n, donde q^2
divide p^2, e logo q^2 divide 1, pois q^2 é primo com p^2). Assim, n=p^2, e a
raiz de n é igual a p, que é inteiro.

   Abraços,
 Gugu



This message was sent using IMP, the Internet Messaging Program.

=
Instruções para entrar na lista, sair da lista e usar a lista em
http://www.mat.puc-rio.br/~nicolau/olimp/obm-l.html
=


Re: [obm-l] Questãozinha q tá me dando dor de cabeça - Calculo 1 - Exponencial de Matrizes

2006-02-03 Por tôpico gugu
   No B) a propriedade vale sempre: para toda matriz A, podemos achar uma matriz
X complexa invertível com A=XTX^(-1), onde T é triangular inferior com elementos
c_1,c_2,...,c_n na diagonal, os quais são os autovalores de A (por exemplo pela
forma canônica de Jordan). Temos então exp(A)=X.exp(T).X^(-1). Temos que exp(T)
também é triangular inferior, com elementos e^(c_1),
e^(c_2),...,e^(c_n) na diagonal. Assim, seu determinante (que coincide com o
determinante de exp(A)) vale e^(c_1).e^(c_2)e^(c_n)=e^(c_1+c_2+...+c_n)=
=e^(Tr(A)).
   Abraços,
 Gugu


Citando João Vitor [EMAIL PROTECTED]:

 Exponencial de Matrizes

 Dada uma matriz A de ordem n x n, a exponencial de A é definida por

  exp(A) = e^(A) := Somatório de i até infinito de: (Ai)/(i!) = I + A +
 (A^2)/(2!) + ... (A^n)/(n!)...

 A) Calcular a Exponencial de :

   | 0 1|   | 0 1 1 | | 1 0 0 |
 A=  | 0 0| ;   B=  | 0 0 1 |  ;C= |  0 1 0 |
 | 0 0 0 | | 0 0 1 |

 B)Prove que para toda matriz diagonalizável D pentencente M_n(Reais) tem-se
 que:

   det(e^d) = e^(Tr(D))

 onde Tr(D) é o traço da matriz D. Você acha que este resultado é válido para
 toda Matriz?

 Vlw
 Abraços





This message was sent using IMP, the Internet Messaging Program.

=
Instruções para entrar na lista, sair da lista e usar a lista em
http://www.mat.puc-rio.br/~nicolau/olimp/obm-l.html
=


Re: [obm-l] MAIS UM PROBLEMA INTERESSANTE

2006-02-02 Por tôpico gugu
   Caro Klaus,
   Vamos lá:
i) Como o coeficiente líder de f é 1 e o coeficiente constante é 2, as possíveis
raízes racionais de f são 1,-1,2 e -2, as quais não são raízes de f, como se
verifica facilmente. Assim, se f não é irredutível, f(x) pode ser fatorada como
f(x)=(x^2+ax+b)(x^3+rx^2+sx+t), com a,b,r,s,t inteiros. Temos então a+r=-1,
b+ar+s=-4,t+as+br=4,at+bs=0 e bt=2. Agora temos dois casos:
i.1) |b|=2 e |t|=1: at+bs=0 implica que a é par. t+as+br=4 implica que t é par,
absurdo, pois |t|=1.
i.2) |b|=1 e |t|=2: at+bs=0 implica que s é par. t+as+br=4 implica que r é par.
Assim, b+ar+s=-4 implicaria que b é par, absurdo, pois |b|=1.
Isto termina a prova de que f é irredutível.
Abraços,
  Gugu

P.S.: Caso haja dúvidas na afirmação de que f tem duas raízes distintas entre 1
e 2, basta observar que f(1)=f(2)=20 e f(3^(1/2))=5-3.3^(1/2)0.

Citando Klaus Ferraz [EMAIL PROTECTED]:

 Olá mestre,
  nao entendi como provo que o polinomio (x)=x^5-x^4-4x^3+4x^2+2 é um
 polinômio irredutível em Z[x].


 [EMAIL PROTECTED] escreveu:
   Algumas sugestões:
 i) Prove (mais ou menos no braço) que f(x)=x^5-x^4-4x^3+4x^2+2 é um polinômio
 irredutível em Z[x].
 ii) Conclua que, se r^n=a, onde a é racional, para alguma raiz r de f(x)=0
 então
 f(x) divide o polinômio x^n-a, e logo todas as raízes de f têm o mesmo
 módulo.
 Verifique então que f tem duas raízes reais distintas entre 1 e 2, o que nos
 leva a um absurdo.
 Abraços,
 Gugu

 Citando Joÿe3o Silva :

  (OBM - 1995) Mostre que a n-ésima raiz de um número racional (sendo n um
  inteiro positivo) não pode ser raiz do polinômio x^5 - x^4 - 4x^3 + 4x^2 +
 2.
 
 
 
 
  -
  Yahoo! doce lar. Faça do Yahoo! sua homepage.




 
 This message was sent using IMP, the Internet Messaging Program.

 =
 Instruções para entrar na lista, sair da lista e usar a lista em
 http://www.mat.puc-rio.br/~nicolau/olimp/obm-l.html
 =




 -
  Yahoo! doce lar. Faça do Yahoo! sua homepage.





This message was sent using IMP, the Internet Messaging Program.

=
Instruções para entrar na lista, sair da lista e usar a lista em
http://www.mat.puc-rio.br/~nicolau/olimp/obm-l.html
=


Re: [obm-l] Raiz

2006-02-02 Por tôpico gugu
  Caro Klaus,
  Vamos lá:
  Vamos mostrar por indução que, se k é ímpar, (sqrt(2)-1)^k pode ser escrito
como y.sqrt(2)-x=sqrt(2.y^2)-sqrt(x^2), com x e y naturais e 2.y^2-x^2=1 (aqui
N=2.y^2 e N-1=x^2), e, se k é par, (sqrt(2)-1)^k pode ser escrito como
x-y.sqrt(2)=sqrt(x^2)-sqrt(2.y^2), com x e y naturais e x^2-2.y^2=1 (aqui, N=x^2
e N-1=2.y^2). De fato, isso vale para k=1, que é ímpar, com x=y=1.
  Se k é ímpar, e (sqrt(2)-1)^k=y.sqrt(2)-x, com 2.y^2-x^2=1, temos
(sqrt(2)-1)^(k+1)=(sqrt(2)-1)(y.sqrt(2)-x)=(2y+x)-(x+y).sqrt(2); temos k+1 par e
(2x+y)^2-2.(x+y)^2=2.x^2-y^2=1, como queríamos.
  Por outro lado, se k é par e (sqrt(2)-1)^k=x-y.sqrt(2), com x^2-2.y^2=1, temos
(sqrt(2)-1)^(k+1)=(sqrt(2)-1)(x-y.sqrt(2))=(x+y).sqrt(2)-(x+2y); temos k+1 ímpar
e 2.(x+y)^2-(x+2y)^2=x^2-2.y^2=1, e nossa afirmação está provada.
   Abraços,
 Gugu

Citando Klaus Ferraz [EMAIL PROTECTED]:

 Prove que todo numero natural da forma (sqrt(2)-1)^k, k natural, pode ser
 colocado na forma sqrt(N)-sqrt(N-1))


 -
  Yahoo! doce lar. Faça do Yahoo! sua homepage.





This message was sent using IMP, the Internet Messaging Program.

=
Instruções para entrar na lista, sair da lista e usar a lista em
http://www.mat.puc-rio.br/~nicolau/olimp/obm-l.html
=


Re: [obm-l] MAIS UM PROBLEMA INTERESSANTE

2006-01-31 Por tôpico gugu
   Algumas sugestões:
i) Prove (mais ou menos no braço) que f(x)=x^5-x^4-4x^3+4x^2+2 é um polinômio
irredutível em Z[x].
ii) Conclua que, se r^n=a, onde a é racional, para alguma raiz r de f(x)=0 então
f(x) divide o polinômio x^n-a, e logo todas as raízes de f têm o mesmo módulo.
Verifique então que f tem duas raízes reais distintas entre 1 e 2, o que nos
leva a um absurdo.
   Abraços,
  Gugu

Citando Joÿe3o Silva [EMAIL PROTECTED]:

 (OBM - 1995) Mostre que a n-ésima raiz de um número racional (sendo n um
 inteiro positivo) não pode ser raiz do polinômio x^5 - x^4 - 4x^3 + 4x^2 + 2.




 -
  Yahoo! doce lar. Faça do Yahoo! sua homepage.





This message was sent using IMP, the Internet Messaging Program.

=
Instruções para entrar na lista, sair da lista e usar a lista em
http://www.mat.puc-rio.br/~nicolau/olimp/obm-l.html
=


Re: [obm-l] UMA DUVIDA E DOIS PROBLEMAS DA OBM

2006-01-16 Por tôpico gugu
   Vamos lá:
   O comprimento da curva a:[0,2.pi]-R^2 é a integral de 0 a 2.pi de |a´(t)|, e
portanto é menor ou igual a 2.pi.2=4.pi.
   1- O resto da divisão de p(x) (e logo de p^n(x), para todo n) por 101 só
depende do resto da divisão de x por 101. Fazendo x variar em A={0,1,...,100},
definimos g(x) como o resto da divisão de p(x) por 101 (o qual também pertence
a A). Se mostrarmos que g é uma bijeção de A, não é difícil ver que existe n0
tal que g^n(x)=x para todo x em A, pois A é finito (de fato, só há 101! 
bijeções de A, donde na lista Id,g,g^2,...,g^(101!), há duas bijeções g^i e g^j
iguais, com ij, donde g^(j-i)=Id), e isto resolve o problema.
  Mostrar que g é uma bijeção de A dá um certo trabalho (pode ser checado
diretamente calculando p(k)(mod 101) para k=0,1,...,100, usando, se desejar a
recorrência p(k+4)=4p(k+3)-6p(k+2)+4p(k+1)-p(k), ou por meio de manipulações
que começam dividindo p(y)-p(x) por y-x, e terminam usando o fato de que -3 não
é quadrado módulo 101).
   2- A seguinte solução funciona mesmo que troquemos D por um conjunto limitado
qualquer de R^n (ou por um espaço métrico qualquer):
  Suponha por absurdo que existam x e y com |f(y)-f(x)||x-y|. Seja d0 tal
que |f(y)-f(x)|=|x-y|-5d. Cobrimos D por bolas de raio d, usando o menor número
possível delas. Seja k esse número de bolas. Havendo diversas coberturas com k
bolas de raio d, escolhemos uma que tenha centros em x_1,x_2,...,x_k tais que
soma(1=ij=k)(|x_i-x_j|) seja mímima. Como f é sobrejetiva e não aumenta
distâncias, a união das bolas com centro nos f(x_i) e raio d também cobre D
(pois a bola com centro em f(x_i) e raio d contém a imagem por f da bola de
centro x_i e raio d). Temos |f(x_i)-f(x_j)|=|x_i-x_j| pata quaisquer i,j.
Por outro lado, se x pertence à bola com centro em x_r e raio d e y pertence à
bola com centro em x_s e raio d então temos
|f(x_s)-f(x_r)|=|f(x_s)-f(y)|+|f(y)-f(x)|+|f(x)-f(x_r)|=
=|x_s-y|+(|y-x|-5d)+|x-x_r|=d+(|y-x|-5d)+d=|y-x|-3d=
=|y-x_s|+|x_s-x_r|+|x_r-x|-3d=d+|x_s-x_r|+d-3d=|x_s-x_r|-d|x_s-x_r|, donde
soma(1=ij=k)(|f(x_i)-f(x_j)|)=soma(1=ij=k)(|x_i-x_j|), contradizendo a
minimalidade de soma(1=ij=k)(|x_i-x_j|).
Abraços,
  Gugu



Citando Joÿe3o Silva [EMAIL PROTECTED]:

 - Duvida: na solução do problema 6 da OBM - Nivel U - Segunda Fase, que
 aparece na Eureka 22 está escrito: Temos ainda |a'(t)| é menor que ou igual
 a 2 para todo t, donde o comprimento da curva a é menor ou igual a 4pi.
 Alguém poderia me explicar por que isso é válido.

   - Já faz algum tempo que postei os seguintes problemas da obm. Como ainda
 não apareceu nenhuma solução estou postando-os novamente.

 1- (OBM 1996) Seja p(x) o polinomio x^3 + 14x^2 - 2x + 1. Defina p^n(x)
 como
   p(p^(n -1)(x)). Mostre que existe um inteiro N tal que p^N(x) - x é
 divisivel por 101 para todos os inteiros x.

   2- (OBM 2001 - Nivel U) Seja D o conjunto de pontos de R^2 com |p| menor
 que ou igual a 1. Seja f : D = D uma função sobrejetora tal que
   |f(p) - f(q)| é menor que ou igual a |p - q| para quaisquer p, q de D.
 Prove que
   |f(p) - f(q)| = |p - q|.
   (  |(x,y)| = sqrt(x^2 + y^2)  )

   - obs: Uma solução para o problema 2 encontra-se na Eureka 13. No entanto,
 é definida uma função f~ composição de rotação com espelhamento que coincide
 com f nos pontos p, q, -p e -q. O que me garante a existência de tal função?
 Por quê ela é uma bijeção? Existe uma solução alternativa que não utilize tal
 conceito e nem teoria das medidas?



 -
  Yahoo! doce lar. Faça do Yahoo! sua homepage.





This message was sent using IMP, the Internet Messaging Program.

=
Instruções para entrar na lista, sair da lista e usar a lista em
http://www.mat.puc-rio.br/~nicolau/olimp/obm-l.html
=


Re: [obm-l] raizes primitivas...

2006-01-09 Por tôpico gugu
   Caro Diego,
   Isto não é verdade em geral. Por exemplo, 7 é raiz primitiva módulo 5, mas
não é raiz primitiva módulo 25 (sua ordem módulo 25 é 4). O que é verdade é
que, se p é um primo ímpar e a é raiz primitiva módulo p^2 então a é raiz
primitiva módulo p^k para todo k natural. Veja meu artigo na Eureka 2.
   Abraços,
Gugu

Citando diego andres [EMAIL PROTECTED]:

 pra ser mais preciso, a duvida esta nesta parte da soluçao:


   se a eh raiz primitiva modulo p,e pelo o teorema de euler vem:
   a^(p-1)eh congruente a 1 mod p
   a^(p(p-1))eh congruente a 1 mod p^2
   a^((p^2)(p-1))eh congruente a 1 mod p^3
   
   a^((p^k)(p-1))eh congruente a 1 mod p^k

   logo suponhamos que para um k,a ordem de a modulo p^k=j:
   assim, p-1 divide j que divide (p^k)(p-1) existem estes j´s que satisfazem:
   (p-1),(p(p-1)), ((p^2)(p-1)),...,((p^k)(p-1)) mas como eh que  provo que
 soh o,((p^k)(p-1)) que satisfaz a congruencia:  a^((p^k)(p-1))eh
 congruente a 1 mod p^k

 diego andres [EMAIL PROTECTED] escreveu:
 Alguem poderia provar pra mim que se a eh uma raiz primitiva modulo p
 entao a tambem eh uma raiz primitiva de p^w onde a eh incongruente a 1
 modulo p^2.
   Agradecidamente Diego Andrés

 -
   Yahoo! doce lar. Faça do Yahoo! sua homepage.



 -
  Yahoo! doce lar. Faça do Yahoo! sua homepage.





This message was sent using IMP, the Internet Messaging Program.

=
Instruções para entrar na lista, sair da lista e usar a lista em
http://www.mat.puc-rio.br/~nicolau/olimp/obm-l.html
=


Re: [obm-l] PROBLEMA DIFICIL

2006-01-06 Por tôpico gugu
   Caros colegas,
   O enunciado do problema (e o resto da prova da segunda fase do nível U deste
ano estão em http://www.obm.org.br/provas/obm2005/2Fase_Nivelu2005.pdf ou em
http://www.obm.org.br/provas/obm2005/2Fase_Nivelu2005.doc , por exemplo.
   Uma solução é como segue (vou colocar algumas linhas com pontinhos antes...):
.
.
.
.
.
.
.
.
.
.
.
.
.
.
.
.
.
.
.
.
.
.
 Usaremos o fato de que, se a soma de vários números reais em [-1,1] é 0,
então podemos reordená-los de modo que as somas parciais sempre estejam em
[-1,1] (basta que a cada momento colocamos um número dos que faltam que tenha
sinal contrário ao valor atual da soma).
 Podemos supor, reordenando os vetores e girando o plano, se for preciso,
que a soma de vetores da lista que tem o maior módulo possível é
v_1+v_2+...+v_k, a qual é igual a (0,a), com a0; não é difícil ver que as
ordenadas dos v_i para i=k devem ser positivas, pois se algum desses v_i tiver
ordenada negativa simplesmente retiramos esse v_i da lista e aumentamos o módulo
da soma (como a soma total é 0, v_(k+1)+...+v_n=(0,-a), e as ordenadas dos v_i
são negativas). Podemos ainda, reordenando v_1,v_2,...,v_k usando o fato
inicial, supor que, para s=k, a abscissa de v_1+v_2+...+v_s sempre pertença a
[-1,1]. Do mesmo modo, podemos supor que, para k+1=t=n, a abscissa de
v_(k+1)+v_(k+2)+...+v_t também pertence a [-1,1]. Agora usamos de novo o fato
inicial para ordenadas para intercalar os v_i com 1=i=k com os v_j com
k+1=j=n (preservando a ordem dos v_i e dos v_j) de maneira que a ordenada das
somas parciais sempre esteja em [-1,1]. Qualquer uma dessas somas parciais será
a soma de uma parcela do tipo v_1+v_2+...+v_s, com s=k com outra do tipo
v_(k+1)+v_(k+2)+...+v_t, com k+1=t=n, e logo tem abscissa em [-2,2]. Assim,
qualquer soma parcial tem módulo limitado por (2^2+1^2)^(1/2)=5^(1/2).
Abraços,
  Gugu


Citando Valter Rosa [EMAIL PROTECTED]:

 Como eu acho a definição deste problema ?
 Dá pra colocar aqui na lista ?

   - Original Message -
   From: Joÿe3o Silva
   To: obm-l@mat.puc-rio.br
   Sent: Wednesday, December 28, 2005 8:48 PM
   Subject: [obm-l] PROBLEMA DIFICIL


   Alguem sabe como se faz o problema 3 da OBM, nivel universitario, fase 2,
 deste ano (2005) ?


 --
   Yahoo! doce lar. Faça do Yahoo! sua homepage.


 --


   No virus found in this incoming message.
   Checked by AVG Free Edition.
   Version: 7.1.371 / Virus Database: 267.14.8/215 - Release Date: 27/12/2005






This message was sent using IMP, the Internet Messaging Program.

=
Instruções para entrar na lista, sair da lista e usar a lista em
http://www.mat.puc-rio.br/~nicolau/olimp/obm-l.html
=


Re: [obm-l] trigo

2005-11-24 Por tôpico gugu
   É um pouco menos trabalhoso fazer cos(261)=cos(180+81)=-cos(81)=-cos(36+45).
Lembramos que cos(45)=raiz(2)/2 e cos(36)=(raiz(5)+1)/2.
   Abraços,
 Gugu

P.S: Para achar cos(36), podemos usar que cos(36)=1-2sen(18)²=1-2cos(72)^2. Por
outro lado, 2cos(72)=w+w^(-1), onde w=e^(2.pi.i/5) satisfaz w^4+w^3+w^2+w+1=0,
e, dividindo por w^2, temos 0=1+w+1/w+w^2+1/w^2=1+2cos(72)+((2cos(72))^2-2)).


Quoting Fábio Dias Moreira [EMAIL PROTECTED]:

 On Thu, 2005-11-24 at 11:35 -0200, Rodrigo Augusto wrote:
  bom dia gente,
 
  como eu faço pra calcular o cos de 261º?
  [...]

 cos 261 = cos (72 + 72 + 72 + 45). Usando repetidamente a fórmula da
 soma, é fácil obter uma expressão explícita para o valor de cos 261, que
 dependa apenas de somas, subtrações, multiplicações, divisões e
 extrações de raízes quadradas.

 Mas é mais fácil comprar uma calculadora científica de dez contos no
 camelô mais próximo da sua casa.

 []s,

 --
 Fábio Dias Moreira


 =
 Instruções para entrar na lista, sair da lista e usar a lista em
 http://www.mat.puc-rio.br/~nicolau/olimp/obm-l.html
 =






This message was sent using IMP, the Internet Messaging Program.

=
Instruções para entrar na lista, sair da lista e usar a lista em
http://www.mat.puc-rio.br/~nicolau/olimp/obm-l.html
=


[obm-l] VIII Olimpíada Iberoamericana de Matemática Universitária

2005-11-16 Por tôpico gugu
   Para os alunos universitários:
   Venho convidá-los para participar neste sábado, 19/11/2005, às 14:00 hs, nas
universidades cadastradas, da VIII Olimpíada Iberoamericana de Matemática
Universitária. Esta é uma competição da qual participam diversos países da
América Latina e da Península Ibérica. As provas são feitas em cada
universidade e enviadas para a coordenação central (no Brasil, que está
organizando o evento este ano). Vejam mais informações em
http://www.obm.org.br/univ/oimu.htm . A prova do ano passado pode ser
encontrada em http://www.obm.org.br/oimu/ , e provas dos anos anteriores em
http://www.obm.org.br/provas.htm . A lista das universidades cadastradas com
seus respectivos professores responsáveis está em
http://www.obm.org.br/nivelu.htm .
Na cidade de São Paulo a prova também deve ser realizada na coordenação regional
da OBM, no Colégio ETAPA - Rua Nicolau de Sousa Queirós, 330
CEP: 04101-040 - São Paulo-SP Tel: (11)-55740722 Fax: (11)-55740722 R. 168 , sob
a coordenação do Prof. Edmilson Motta (e-mail:[EMAIL PROTECTED]).
Alunos de universidades não cadastradas podem fazer a prova em alguma
universidade cadastrada, para o que devem entrar em contato com o professor
responsável da universidade onde farão a prova. Alunos de universidades
cadastradas devem também entrar em contato com o respectivo professor
responsável para obter informações sobre o local onde deve ser aplicada a
prova.
   Cordialmente,
   Carlos Gustavo Moreira (Gugu)



This message was sent using IMP, the Internet Messaging Program.

=
Instruções para entrar na lista, sair da lista e usar a lista em
http://www.mat.puc-rio.br/~nicolau/olimp/obm-l.html
=


Re: [obm-l] PELO SIM, PELO

2005-09-15 Por tôpico gugu
   Oi Nicolau,
   Você está sendo coerente, mas é possível interpretar de outra maneira - se a
resposta de um mentiroso compulsório consta de várias afirmações, todas elas
devem ser falsas. Na verdade eu acho esse problema meio mal formulado por isso
mesmo. As perguntas (e as respostas) podem ser arbitrariamente complexas ?
   Abraços,
Gugu

Quoting Nicolau C. Saldanha [EMAIL PROTECTED]:

 On Wed, Sep 14, 2005 at 05:32:06PM -0300, Carlos Gustavo Tamm de Araujo
 Moreira wrote:
 Oi Nicolau,
 Mas se eu perguntar a ele (e isso, nessa interpreta??o,  ? uma pergunta
 s?):
  diga sobre cada um de voc?s se ? honesto ou n?o (na verdade a pergunta da
  minha solu??o n?o ? bem essa, mas algo como se eu perguntasse a voc? sobre
  se cada um de voc?s ? honesto ou n?o, o que voc? responderia em cada caso
 ?)
  - ele decidir? como quiser no in?cio se vai mentir sobre todos ou se vai
 dizer
  a verdade sobre todos. A quest?o ? se valem perguntas compostas ou n?o...

 Não entendi nada. O que você quer dizer com uma pergunta composta?
 Você está fazendo UMA pergunta, e se a resposta não for a correta,
 isto conta como uma mentira.

 A sua pergunta para mim pode ser reformulada assim:

 A cada uma destas pessoas associe uma letra de uma cadeia de letras:
 esta letra é H se o cara for honesto e D caso contrário.
 Se eu perguntasse a você qual a cadeia correta, o que você me responderia?

 Há uma resposta certa e um monte de respostas erradas e isso arruina o truque
 pois mudar duas vezes não necessariamente traz você de volta ao mesmo lugar.

 []s, N.
 =
 Instruções para entrar na lista, sair da lista e usar a lista em
 http://www.mat.puc-rio.br/~nicolau/olimp/obm-l.html
 =






This message was sent using IMP, the Internet Messaging Program.

=
Instruções para entrar na lista, sair da lista e usar a lista em
http://www.mat.puc-rio.br/~nicolau/olimp/obm-l.html
=


Re: RES: [obm-l] conjunto de irracionais fechado com relacao aa soma

2005-09-14 Por tôpico gugu
   Caro Artur,
   S tem que ser um conjunto de seqüências limitadas de naturais, não ? Vale a
pena supor que são seqüências de inteiros positivos, ou pelo menos que têm
infinitos termos não nulos. Dá para provar que nesse exemplo os elementos de A
são transcendentes por serem números de Liouville (todos os números de
Liouville são transcendentes). Um número de Liouville é um real c tal que para
todo n existem infinitos racionais p/q com |c-p/q|1/q^n. Um número algébrico
nunca é de Liouville: se f é um polinômio de grau n com coeficientes inteiros
tal que f(c)=0, existe K0 tal que |f´(x)|=K para todo x em [c-1,c+1]. Por
outro lado, se p/q é diferente de c e está muito perto (em particular a uma
distância menor que 1) de c então f(p/q) é um racional não nulo que pode ser
escrito como um inteiro dividido por q^n, e logo |f(p/q)|=1/q^n. Por outro
lado, pelo teorema do valor médio, existe d entre c e p/q com
f(p/q)=f(p/q)-f(c)=f´(d).(p/q-c), donde
1/q^n=|f(p/q)|=|f(p/q)-f(c)|=|f´(d)|.|p/q-c|=K.|p/q-c|. Assim, temos sempre
|c-p/q|=1/(K.q^n), e não existem, por exemplo, infinitos racionais p/q com
|c-p/q|1/q^(n+1) (pois teríamos necessariamente |q|K, além de |c-p/q|1).
Os elementos de A no exemplo são números de Liouville, pois, se todos os a(k)
são inteiros positivos limitados por K, somando a série até o n-ésimo termo,
obtemos um racional com denominador 2^(n!) cuja distância ao valor da série
infinita é positiva é menor que 2.K/2^((n+1)!), que é muito menor que
(1/2^(n!))^n.
   Eu tinha outro exemplo (que, pensando bem, não é tão diferente assim desse):
o conjunto A dos números que, quando escritos em base 2, têm infinitos bits
iguais a 1, mas, se f(n) é o número de bits iguais a 1 dentre os n primeiros
bits de um elemento de A, então f(n)=o(n), isto é, lim(n-infinito)(f(n)/n)=0.
   Abraços,
 Gugu


   Acho que eu tenho outro exemplo: seja
Quoting Artur Costa Steiner [EMAIL PROTECTED]:

 Nesta solucao, a base apontada pelo Daniel eh o que se chama de base de
 Hamel?
 Nenhuma base de R sobre Q popde ser enumeravel, certo?

 Sobre este assunto, especificamente sobre a questao levantada pelo Nicolau,
 no sentido de efetivamente construirmos o conjunto, eu encontrei na internet
 uma solucao proposta por um matematico americano, que nao sei dizer dizer se
 eh correta:


 Sendo S a colecao de todas as sequencias limitadas de R, entao, segundo o
 matematico, o conjunto A = {Soma(k=1 a oo) a(k)/(2^(k!) | sequencia {a(k)}
 pertence a S} satisfaz ao desejado. Eh facil ver que A nao eh enumeravel  e
 que eh fechado com relacao aa soma. O matematico afirma que o Liouville's
 Approximation Theorem , Teorema da Aproximação de Liouvile,  implica que os
 elementos de A sejam transcendentes, logo irracionais (nao foi apresentada
 prova desta afirmacao). Se o matematico estiver certo, entao temos de fato
 um conjunto construido explicitamente sem recorrer ao axioma da escolha.

 Artur







 -Mensagem original-
 De: [EMAIL PROTECTED] [mailto:[EMAIL PROTECTED]
 nome de Nicolau C. Saldanha
 Enviada em: sexta-feira, 12 de agosto de 2005 09:22
 Para: obm-l@mat.puc-rio.br
 Assunto: Re: [obm-l] conjunto de irracionais fechado com relacao aa soma


 On Thu, Aug 11, 2005 at 09:28:31PM -0300, [EMAIL PROTECTED] wrote:
   ''Encontre um conjunto de irracionais que nao seja enumeravel e seja
 fechado
   ''com relacao aa soma
 
  Observe que se V é um espaço vetorial de dimensão enumerável sobre Q
  (racionais), então V é isomorfo ao espaço dos polinômios em uma variável
  sobre Q, e,
  portanto, V é enumerável. Em particular, a dimensão de R (reais) sobre Q
  é não-enumerável.
 
  Seja B* uma base de R sobre Q contendo o 1, e seja B = B*\{1}.

 Esta solução (correta) usa o axioma da escolha para obter a base e portanto
 o conjunto obtido no final não é dado explicitamente.

 Uma pergunta que eu nao sei reponder:

 É possível responder a pergunta original com a interpretação de que
 encontre significa construa ou descreva explicitamente
 (sem usar o axioma da escolha)?

 []s, N.

 =
 Instruções para entrar na lista, sair da lista e usar a lista em
 http://www.mat.puc-rio.br/~nicolau/olimp/obm-l.html
 =

 =
 Instruções para entrar na lista, sair da lista e usar a lista em
 http://www.mat.puc-rio.br/~nicolau/olimp/obm-l.html
 =






This message was sent using IMP, the Internet Messaging Program.

=
Instruções para entrar na lista, sair da lista e usar a lista em
http://www.mat.puc-rio.br/~nicolau/olimp/obm-l.html
=


Re: [obm-l] PELO SIM, PELO NÃO!

2005-09-14 Por tôpico gugu
  Caro Jorge Luis,
  Tem uma solução mais ou menos clássica com uma pergunta só: escolha um cara
qualquer e pergunte:Se eu perguntasse a você sobre cada uma dessas 5 pessoas
(incluindo você) se são honestas ou não, o que você responderia ?  Se nesse
momento ele for dizer a verdade, vai indicar o honesto corretamente. Se ele for
 mentir sistematicamente nessa resposta, ele responderia errado sobre todas as
pessoas, caso perguntado diretamente sobre se são honestas ou não. Como ele é
perguntado sobre o que responderia, ao mentir nessa resposta ele diria a
informação correta. Assim, de qualquer jeito você obteria a informação correta.
   É claro que você pode achar que essa solução está meio roubada, e que devemos
colocar restrições sobre que tipo de pergunta pode ser feito - por exemplo, o
problema é diferente se só admitirmos perguntas cujas possíveis respostas sejam
sim ou não.
   Abraços,
 Gugu

Quoting Jorge Luis Rodrigues e Silva Luis [EMAIL PROTECTED]:

 Perdão! Nicolau e demais colegas pela suposta arrogância que não houve
 intenção de provocação. Quanto ao desenho anexo no enunciado do problema,
 constam 12 quadrados (quarteirões) com suas 4 ruas horizontais e 5 ruas
 verticais. E aí vem a inevitável pergunta: Se vocês fossem da comissão do
 vestibular da FGV-SP, que resposta considerariam como a correta: 10, 20, 35,
 ?

 Cinco pessoas estão em uma sala. Uma delas é um sujeito honesto, que sempre
 diz sempre a verdade. As outras quatro alternam uma mentira e uma verdade e
 podem começar por qualquer uma das duas. Todos sabem quem é o sujeito
 honesto, menos você. Qual é o número mínimo de perguntas necessário para
 descobrir o honesto?

 NOTA: Achei muito simpático o termo nosso herói ...  Abraços!

 _
 MSN Messenger: converse online com seus amigos .
 http://messenger.msn.com.br

 =
 Instruções para entrar na lista, sair da lista e usar a lista em
 http://www.mat.puc-rio.br/~nicolau/olimp/obm-l.html
 =






This message was sent using IMP, the Internet Messaging Program.

=
Instruções para entrar na lista, sair da lista e usar a lista em
http://www.mat.puc-rio.br/~nicolau/olimp/obm-l.html
=


Re: [obm-l] PELO SIM, PELO NÃO!

2005-09-14 Por tôpico gugu
   Caro Nicolau,
   Eu concordo que a minha solução admite críticas nessa linha, mas o Elon, por
exemplo, argumenta do mesmo jeito sobre um problema análogo num livro dele
(supondo que um cara que vai mentir numa resposta mente sobre tudo). De
qualquer jeito eu acho que a melhor conclusão é que o problema está mal
formulado, e seria melhor especificar que tipo de pergunta pode ser feita.  Por
outro lado, eu não entendi bem a sua última objeção: se os desonestos
decidiremhttp://www.impa.br/biblioteca/index.html, em função da minha pergunta,
entre dizer a verdade em toda a sua resposta ou só fazer afirmações falsas em
sua resposta então a minha solução funciona. Seria interessante saber a opinião
do Jorge Luiz sobre se as respostas devem ser sempre sim ou não e sobre se
mentirosos mentem consistentemente ou podem mentir parcialmente no caso em que
respostas mais complicadas sejam admissíveis.
   Abraços,
 Gugu

Quoting Nicolau C. Saldanha [EMAIL PROTECTED]:

 On Wed, Sep 14, 2005 at 01:54:35PM -0300, [EMAIL PROTECTED] wrote:
Caro Jorge Luis,
Tem uma solução mais ou menos clássica com uma pergunta só: escolha um
 cara
  qualquer e pergunte:Se eu perguntasse a você sobre cada uma dessas 5
 pessoas
  (incluindo você) se são honestas ou não, o que você responderia ?  Se
 nesse
  momento ele for dizer a verdade, vai indicar o honesto corretamente. Se ele
  for mentir sistematicamente nessa resposta, ele responderia errado sobre
  todas as pessoas, caso perguntado diretamente sobre se são honestas ou não.
  Como ele é
  perguntado sobre o que responderia, ao mentir nessa resposta ele diria a
  informação correta. Assim, de qualquer jeito você obteria a informação
  correta.

 Eu discordo desta interpretação. Digamos que os candidatos estejam arrumados
 assim: d,d,h,d,d (onde h é honesto e d não) e que você se faça esta pergunta
 ao primeiro da fila. Mesmo se interpretarmos que ele já decidiu que é hora
 de mentir e que perguntado diretamente ele responderia h,h,d,h,h, ele pode
 responder, por exemplo, h,d,h,h,h: ele estará mentindo e você não descobriu
 nada (ou tira a conclusão errada).

 Acho que esta solução se aplica a perguntas com resposta Sim ou Não
 e mesmo assim não tenho certeza se se aplica a este problema. Não entendo
 que o enunciado deixe claro que exista uma hora de mentir predeterminada
 antes de você formular a primeira pergunta. Ou seja, os desonestos podem
 decidir se vão mentir ou não na primeira pergunta em função da pergunta,
 arruinando este truque.

 []s, N.
 =
 Instruções para entrar na lista, sair da lista e usar a lista em
 http://www.mat.puc-rio.br/~nicolau/olimp/obm-l.html
 =






This message was sent using IMP, the Internet Messaging Program.

=
Instruções para entrar na lista, sair da lista e usar a lista em
http://www.mat.puc-rio.br/~nicolau/olimp/obm-l.html
=


Re: [obm-l] valor medio para integrais

2005-09-08 Por tôpico gugu
   Caro Eric,
   As condições 3 e 4 não parecem necessárias (pelo menos desde que a área de B
seja finita): Se area(B)=0 qualquer P serve. Se area(B)0, seja
m=integral(f dxdy sobre B)/area(B). Temos integral((f-m)dxdy sobre B)=0, donde
existem Q e R em B tais que (f-m)(Q)=f(Q)-m=0 e (f-m)(R)=f(R)-m=0. Ligando P
a Q por uma curva contínua, e usando o teorema do valor intermediario em R para
a composta de f com essa curva, segue que existe algum ponto P na curva com
f(P)=0.
   A condição 6 serve para que a área de B (e a integral) estejam bem definidas,
se estivermos trabalhando com a integral de Riemann. Se estivermos trabalhando
com a integral de Lebesgue, que é mais geral, ela também não parece necessária,
e o argumento acima parece funcionar também.
   Abraços,
 Gugu

Quoting Eric Campos [EMAIL PROTECTED]:

 Saudacoes!

 Estou com uma duvida com respeito ao topico
 propriedade do valor medio para integrais, do livro
 do Guidorizzi (vol.3, 2.ed). O teorema do livro eh o
 seguinte:

 Seja f:B-R, B contido em R^2. Cumprindo-se:

 (hipotese)
 1. f integravel;
 2. f eh continua em B;
 3. B eh limitado;
 4. B eh fechado;
 5. para cada par de pontos b_1, b_2 em B, existe uma
 curva continua (com imagem contida em B) que liga
 ambos;
 6. a fronteira de B tem conteudo nulo (um conjunto D
 tem conteudo nulo se para cada eps0 existe um numero
 finito de retangulos A_1, A_2, ... A_n, tais que D
 esteja contido na uniao dos A_i's e a soma das areas
 destes A_i's eh menor que eps.)

 entao

 (tese)
 existe pelo menos um ponto P=(r,s) em B tal que

 integral(f dxdy sobre B) =
 = f(r,s)*integral(dxdy sobre B) =
 = f(r,s)*area(B).

 Gostaria de saber se a tese continua valendo se
 omitirmos alguma das 6 condicoes da hipotese.

 Eh claro que a condicao 1 tem que valer, senao a tese
 nao faria sentido.

 Tambem sei que a condicao 2 (continuidade) tem que
 valer. Basta considerar o exemplo:

 f(x,y) = 1 se x=0 e f(x,y) = x caso contrario, para
 (x,y) satisfazendo -1=x=1, -1=y=1, isto eh, B eh o
 quadrado de vertices (-1,-1), (-1,1), (1,1), (1,-1).
 Neste caso integral(f dxdy sobre B) = 0, mas nao
 existe (r,s) em B tal que f(r,s)=0.

 Eh facil ver que a condicao 5 eh necessaria. Tome B =
 B_1 uniao B_2 onde a distancia entre B_1 e B_2 eh
 maior que zero; defina f(x) = -1 se x estah em B_1 e
 f(x)=1 se x esta em B_2 (x eh ponto do plano). Admita
 ainda que as areas de B_1 e B_2 sejam iguais. Entao a
 integral de f sobre B eh 0, mas nao existe x em B tal
 que f(x)*Area(B)=0, pois ou f(x)=1 ou f(x)=-1. Logo a
 condicao 5 eh necessaria.

 Entao cada uma das condicoes 1, 2, e 5 eh necessaria,
 porem nao consegui exemplos que me mostrassem que as
 condicoes 3, 4 e 6 sao tambem necessarias. Voces
 poderiam me dar um exemplo para cada um desses 3
 casos?

 Amplexos!

 Eric.



 ==
 http://geocities.yahoo.com.br/mathfire2001
 www.mathfire.pop.com.br
 Enciclopedia de Matematica - Aulas
 Formulas para primos - Grupos de Estudo
 Projeto Matematica para Todos
 [EMAIL PROTECTED]
 MSN: [EMAIL PROTECTED]
 ==

 __
 Converse com seus amigos em tempo real com o Yahoo! Messenger
 http://br.download.yahoo.com/messenger/
 =
 Instruções para entrar na lista, sair da lista e usar a lista em
 http://www.mat.puc-rio.br/~nicolau/olimp/obm-l.html
 =






This message was sent using IMP, the Internet Messaging Program.

=
Instruções para entrar na lista, sair da lista e usar a lista em
http://www.mat.puc-rio.br/~nicolau/olimp/obm-l.html
=


Re: [obm-l] 25o Colóquio Brasileiro de Matemática

2005-05-24 Por tôpico gugu
   Caros colegas,
   O colóquio deste ano (de 24 a 29 de julho) vai ter, pela primeira vez, uma
sessão de olimpíadas, que eu estou coordenando, com palestras temáticas,
sessões de problemas e uma mesa redonda. Os participantes (na maioria alunos
universitários) podem solicitar bolsas-auxílio para participarem do colóquio.
Para isso devem se inscrever no evento ATÉ 3 DE JUNHO. Peço aos interessados na
sessão de olimpíadas que queiram se inscrever no colóquio que me escrevam (para
[EMAIL PROTECTED]) (algumas dessas bolsas-auxílo devem estar ligadas à sessão de
olimpíadas). Vejam instruções sobre inscrição e pedidos de bolsa em
http://www.impa.br/pesquisa/pesquisa_coloquio_brasileiro_de_matematica/25_coloquio_instrucao_inscricao.pdf
A página principal do colóquio é
http://www.impa.br/pesquisa/pesquisa_coloquio_brasileiro_de_matematica/25_coloquio.html

O texto abaixo esta´ em
http://www.impa.br/pesquisa/pesquisa_coloquio_brasileiro_de_matematica/25_coloquio_olimpiada.html
 :

Olimpíada de Matemática

Esta sessão tratará de diversos temas relevantes para as Olimpíadas de
Matemática, incluindo um panorama das Olimpíadas Brasileiras e Internacionais.
As principais atividades previstas são:

- Mesa Redonda
- Palestras (teoria e problemas) sobre: Combinatória, Teoria Elementar dos
Números, Geometria e Análise.
- Oficina de Problemas (serão discutidas algumas das provas recentes das
Olimpíadas Brasileiras e Internacionais)

Estudantes e matemáticos de todas as idades são bem-vindos.

Coordenação: Carlos Gustavo Moreira, IMPA

   Abraços,
  Gugu




Citando Domingos Jr. [EMAIL PROTECTED]:

 Rafael Castilho wrote:

 Alguém vai/pretende ir ao colóquio em julho? Preciso de algumas
 informações.
 
 
 
 Eu vou... talvez seja melhor você procurar o pessoal do IMPA para tirar
 suas dúvidas.
 =
 Instruções para entrar na lista, sair da lista e usar a lista em
 http://www.mat.puc-rio.br/~nicolau/olimp/obm-l.html
 =






This message was sent using IMP, the Internet Messaging Program.

=
Instruções para entrar na lista, sair da lista e usar a lista em
http://www.mat.puc-rio.br/~nicolau/olimp/obm-l.html
=


[obm-l] Re: Fwd: [obm-l] GMAT / Dúvidas .

2003-10-23 Por tôpico gugu
   Caro Marcos,
   Na verdade eu vi a sua mensagem, mas eu nao sei o que e` GMAT... O que e`
isso, e` uma especie de concurso ? Nesse caso voce sabe onde se pode encontrar
questoes desse GMAT ? Eu acho que o pessoal nao respondeu por ignorancia, como eu...
   Abracos,
 Gugu

Quoting Marcos Braga [EMAIL PROTECTED]:

 
 
 Caramba !! Fui totalmente ignorado , ninguém respondeu ...
 
 Tá bom , não pergunto mais ...:))
 
 Mesmo assim se alguma alma caridosa puder me respoder ficarei muito feliz.
 
 Marcos .
 
 
 X-Sender: [EMAIL PROTECTED]
 X-Mailer: QUALCOMM Windows Eudora Version 5.2.1
 Date: Wed, 22 Oct 2003 18:20:56 -0200
 To: [EMAIL PROTECTED]
 From: Marcos Braga [EMAIL PROTECTED]
 Subject: [obm-l] GMAT
 X-MIME-Autoconverted: from quoted-printable to 8bit by 
 sucuri.mat.puc-rio.br id RAA14433
 Sender: [EMAIL PROTECTED]
 Reply-To: [EMAIL PROTECTED]
 
 Oi Galera ,
 
 Sou novo na lista e uma apaixonado por Matemática e Filosofia . Com 
 certeza meu conhecimento de matemática não é tão bom como de vcs, e sendo 
 assim prometo não fazer perguntas idiotas . :))
 
 Estou para prestar uma prova no estilo GMAT , alguém conhece alguma 
 literatura , em português se possível, com características das questões GMAT
 ?
 
 Abraços .
 
 Marcos .
 
 
 =
 Instruções para entrar na lista, sair da lista e usar a lista em
 http://www.mat.puc-rio.br/~nicolau/olimp/obm-l.html
 =
 
 
 =
 Instruções para entrar na lista, sair da lista e usar a lista em
 http://www.mat.puc-rio.br/~nicolau/olimp/obm-l.html
 =
 




-
This mail sent through IMP: http://horde.org/imp/
=
Instruções para entrar na lista, sair da lista e usar a lista em
http://www.mat.puc-rio.br/~nicolau/olimp/obm-l.html
=


Re: [obm-l] limite de sin(n)^n

2003-10-23 Por tôpico gugu
   Oi Salvador,
   Voce tem toda razao: limsup(sen(n)^n)=1. De fato, pelo menos um entre cada 
dois p_n consecutivos e' impar (segue de p_(-1)=1, p_(n+2)=p_n (mod p_(n+1)). 
Assim, se p_n e' impar e p_(n+1) e' par entao p_(n+2) e' impar. Agora, se p_n 
e' impar mas e' 3 mod 4 entao 3.p_(n+1) e' 3 mod 4 e |2/pi-(3.p_n)/(3.q_n)|=
=|2/pi-p_n/q_n|1/(q_n)^2, e logo |3.q_n-(3.p_n).pi/2|3.pi/(2.q_n), e voce 
pode concluir do mesmo jeito.
Para quem nao sabe direito do que a gente esta' falando, leiam meu 
artiguinho sobre fracoes continuas na Eureka 3...
Abracos,
 Gugu

P.S.: Acho que da' para mostrar tambem que o liminf e' -1, mas certamente para 
a maioria dos valores de n a sequencia fica bem perto de 0...

Quoting Salvador Addas Zanata [EMAIL PROTECTED]:

 
 Caro Claudio,
 
 Essa problema eh f...
 
 
 Para que sin(n)^n de problema, temos que escolher um n
 tal que
 
 
   |n-(pi/2+2pik)| seja pequeno. Isso eh equivalente a:
 
 
   |2/pi.n-(1+4k)| seja pequeno. Como 2/pi eh
 irracional, se existirem
 
 convergentes pn/qn de 2/pi, tais que pn = 1+4kn,
 entao,
 
 
   |2/pi.qn-(1+4kn)|1/qn.
 
 
 
 Aqui vou fazer uma hipotese perigosa, que nao pensei
 se eh verdade. Vamos supor
 que existem infinitos convergentes tais que
 pn == 1 mod 4.
 
 
 Isto vai implicar, fazendo umas majoracoes chatas, que
 sin(qn) eh aprox. igual a
 (1-c/qn^2), para um c real que nao depende de n.
 
 Assim, (sin(qn))^qn ~= (1-c/qn^2)^qn, que me parece
 que vai a 1.
 
 Nao conferi todos os passos, muito menos sei se a
 hipotese sobre os convergentes eh
 verdade, mas parece que esse limite nao existe.
 
 
 Abraco,
 
 Salvador
 
 
 
 
 =
 Instruções para entrar na lista, sair da lista e usar a lista em
 http://www.mat.puc-rio.br/~nicolau/olimp/obm-l.html
 =
 




-
This mail sent through IMP: http://horde.org/imp/
=
Instruções para entrar na lista, sair da lista e usar a lista em
http://www.mat.puc-rio.br/~nicolau/olimp/obm-l.html
=


Re: [obm-l] Elevador

2003-07-18 Por tôpico gugu
 Oi Nicolau,
 Acho que consegui uma solucao com 14 andares: e´ que P:=P2(Z/(2)) tem 7 
elementos e 7 retas (seja R o conjunto de suas retas). Tomamos como conjunto 
dos andares Px{0,1}, e como conjunto dos elevadores a diagonal de RxR.
Da´ para escrever mais explicitamente isso: os andares sao 1,2,...,14.
Os elevadores sao {1,2,3,8,9,10},{1,4,5,8,11,12},{1,6,7,8,13,14},
{2,4,6,9,11,13},{2,5,7,9,12,14},{3,4,7,10,11,14} e {3,5,6,10,12,13}.
 Abracos,
  Gugu
 
Citando Nicolau C. Saldanha [EMAIL PROTECTED]:

 On Wed, Jul 16, 2003 at 09:03:30PM -0300, [EMAIL PROTECTED] wrote:
  Num prédio de apartamentos há 7 elevadores que param
  em não mais que 6 andares. 
  É possível ir de um andar a qualquer outro sem trocar de elevador. Qual é o
 
  número máximo de andares que esse prédio pode ter?   (RPM/IME/USP)
 
 O problema est bem legal. Ainda nao completei uma solucao mas vou mandar
 meus resultados parciais.
 
 N = 14
 
 De fato, temos no maximo 42 paradas (uma parada est um par (i,j) onde
 i est um andar, j est um elevador, e o elevador j para no andar i).
 Se N  11 devemos ter pelo menos tres paradas por andar
 (pois duas paradas nos dao apenas dois elevadores que atingem
 no maximo 10 outros andares). Assim 3N = 42 e N = 14.
 
 N = 12
 
 Basta observar a configuracao abaixo onde um + indica que o elevador
 para naquele andar e um . indica que ele nao para. Os andares estao 
 um em cima do outro, claro, e os elevadores um ao lado do outro.
 Observe que 6 elevadores foram suficientes.
 
 +++...
 +++...
 +++...
 +..++.
 +..++.
 +..++.
 .+.+.+
 .+.+.+
 .+.+.+
 ..+.++
 ..+.++
 ..+.++
 
 Eu fiz uns diagramas e me convenci que N = 13 mas a prova foi
 meio bracal, no caso a caso, e pode estar errada.
 
 []s, N.
 
 =
 Instruções para entrar na lista, sair da lista e usar a lista em
 http://www.mat.puc-rio.br/~nicolau/olimp/obm-l.html
 =
 




-
This mail sent through IMP: http://horde.org/imp/
=
Instruções para entrar na lista, sair da lista e usar a lista em
http://www.mat.puc-rio.br/~nicolau/olimp/obm-l.html
=


Re: [obm-l] IMO - Problema 2

2003-07-17 Por tôpico gugu
Caro Paulo,
Usualmente o termo equacao de Pell se refere ao caso j=1 (e o coeficiente 
de b^2 nao depende nem de a nem de b). Nao entendi como concluir uma solucao na 
linha que voce propos. Por outro lado eu consegui (depois de tropecar um pouco) 
achar uma solucao, que reproduzo abaixo, depois de algum espaco, para nao 
atrapalhar quem queira pensar mais no problema.

...



...



...



...



...



...



Vamos la':

   Se b=1 o problema e' achar todos os a tais que a^2/2a=a/2 e' inteiro. Isso 
nos da' as solucoes {(a,1),a par}. Vamos supor agora b=2.
   Se 2ab^2-b^3+1 divide a^2 entao tambem divide a^2.(2b^2)-a(2ab^2-b^3+1)=
=a(b^3-1) e (1-b^3)(2ab^2-b^3+1)+(2b^2)(a(b^3-1))=(1-b^3)^2. Sejam entao 
d=mdc(a,1-b^3), a=kd, 1-b^3=ud. Temos que mdc(k,u)=1 e que 2ab^2-b^3+1 divide 
mdc(a^2,(1-b^3)^2)=d^2, ou seja, d(2kb^2+u) divide d^2, e logo 2kb^2+u divide 
d. Portanto, tambem temos que 2kb^2+u divide b(2kb^2+u)+2kud=ub+2k. Temos agora 
dois casos:
   i)d=kb^2. Entao |u|=|(1-b^3)/d|  b^3/(kb^2)=b/k. Nesse caso, 
|2kb^2+u|=2kb^2-|u|  2kb^2-b/k, enquanto |ub+2k|  b^2/k+2k. Como b=2, 2kb^2-
b/k=kb^2+b(kb-1/k)=b^2/k+2(2k-1/k)=b^2/k+2k, donde |2kb^2+u|  |ub+2k|, e 
portanto devemos ter ub+2k=0, donde b(1-b^3)+2a=dub+2dk=0, e logo a=b(b^3-1)/2.
Isso nos da' a^2/(2ab^2-b^3+1)=b^2/4, que e' inteiro quando b e' par. Isso nos 
da' (todas) as solucoes nesse caso i): {(b(b^3-1)/2,b), b par}.
   ii)d  kb^2. Aqui, como 2kb^2+u divide d, devemos ter kb^2|d|=|2kb^2+u|=
=2kb^2-|u|, donde |u|kb^2. Assim, temos ab^2=kb^2d  |ud|=b^3-1  b^3, donde 
a  b, ou seja, b=a+1. Como 2ab^2-b^3+1 e' congruente a 1 modulo b^2, ou 
2ab^2-b^3+1=1 ou |2ab^2-b^3+1|=|1-b^2|=b^2-1=(a+1)^2-1=a^2+2a  a^2, mas, 
nesse caso, 2ab^2-b^3+1 nao pode dividir a^2. Assim, devemos ter 2ab^2-b^3+1=1, 
donde 2ab^2=b^3, e b=2a. Isso nos da' as solucoes do caso ii): {(a,2a)}.

   Conclusao: as solucoes do problema sao dadas por: {(a,1), a inteiro positivo 
par}, {(b(b^3-1)/2,b), b inteiro positivo par} ou {(a,2a), a inteiro positivo}.
   Abracos,
Gugu

Quoting Paulo Santa Rita [EMAIL PROTECTED]:

 Ola Cicero e demais
 colegas desta lista ... OBM-L,
 
 A sua ideia e uma observacao valida, mas parece-me que o problema exige um 
 tratamento maior... Com efeito, se em :
 
 F(a,b) = a^2/(2ab^2  - b^3 + 1)
 
 fizermos a^2 = 2ab^2 - b^3 + 1 e olharmos para esta desigualdade como uma 
 ineguacao do 2 grau em a, teremos UMA CONDICAO NECESSARIA para que F(a,b)
 
 seja um inteiro, entretanto, esta condicao nao e SUFICIENTE, pois numa 
 fracao P/Q podemos ter P = Q e, no entanto, P/Q nao ser inteiro. Por 
 exemplo : 8/5. Todavia, e muito bom que voce pense na questao. E todos os 
 estudantes, sobretudo os olimpicos, devem seguir o seu exemplo.
 
 Vou te dar uma linha de pensamento para voce explorar.
 
 Vamos colocar a funcao F(a,b) da seguinte forma :
 
 F(a,b) = a^2 / [ (2a - b)*b^2 + 1 ]. Fazendo  G(a,b) = 2a - b, segue que :
 F(a,b) = a^2 / [ G(a,b)*b^2 + 1 ]
 
 PARA TODO i inteiro, a equacao G(a,b)=i   =   2a - b = i tem uma 
 infinidade de solucoes inteiras, pois MDC(2,-1) = 1 divide i, qualquer que
 
 seja i. Mais que isso, para todo i, dado  que
 (a,b)=(0,- i) e uma solucao particular, TODAS as solucoes de G(a,b) = i 
 serao da forma :
 
 a = - t,  b= - i - 2t , t um inteiro qualquer
 
 Conforme voce deve saber do estudo da equacao diofantina AX + BY = C.
 
 E importante observar que procedendo assim EXAURIMOS TODAS AS POSSIVEIS 
 SOLUCOES, pois, qualquer que sejam os inteiros a e b imaginaveis, 2a - b
 
 e tambem um inteiro, isto e, existe um inteiro i tal que 2a - b = i  e, 
 consequentemente, os inteiros a e b que imaginamos pertencerao a alguma
 
 das infinitas equacoes 2a - b = i. E igualmente importante observar que i
 
 e diferente de k entao os cnjuntos solucoes de 2a - b = i e de 2a - b= k 
 sao disjuntos, pois as retas b = 2a - i e
 b=2a - k sao paralelas, se i for diferente de k.
 
 Bom, fixado o que eu disse acima, seja 2a - b = i. Da infinidade de pares 
 (a,b) que satisfazem a esta equacao, procuramos aqueles para os quais :
 
 a^2/(i*b^2 + 1) = j ,  j um inteiro.
 
 Vamos colocar esta equacao assim :
 
 a^2 = i*j*b^2 + j   =  a^2 - (i*j)*b^2 = j
 
 E entao ? Esta reconhecendo a equacao acima ? Creio que sim. Afinal, ela e 
 famosissima : E a conhecidissima EQUACAO DE  PELL !
 
 Bom, Voce deve conhecer os fatos basicos sobre a equacao de Pell. E so 
 concatenar inteligentemente o que voce sabe que a solucao sai serena e 
 tranquila. E aqui eu te deixo so, pra voce continuar ...
 
 
 ABRE PARENTESES
 
 O estudo das equacoes diofantinas, da EQUACAO DE PELL em particular, e um 
 dos acontecimentos mais emocionantes na vida de um estudante de Matematica.
 
 Voce vai ficando chateado de nao encontrar ideias novas e, de repente, se 
 defronta com esta equacao, que traz novidades e surpresas impares, que em 
 muito se afastam da mediocridade e rotina de outros temas. Esta equacao e 
 quase um revigorante intelectual, que devemos

Re: [obm-l] Sequencias

2003-07-17 Por tôpico gugu
   Outro contra-exemplo simpatico para o item c) e' x_k=cos(raiz(k)).
   Abracos,
   Gugu

Quoting Salvador Addas Zanata [EMAIL PROTECTED]:

 
 Pessoal,
 
 Disse bobagem no item c).
 
 
 Obrigado pela correcao, Manoel.
 
 
 Segue o e-mail dele abaixo com a correcao.
 
 
 
 Mais uma vez obrigado ao Manoel.
 
 
 
 Um abraco,
 
 
 Salvador
 
 
 
 On Wed, 16 Jul 2003, Manuel Valentim Pera wrote:
 
  Salvador,
 
 Mande um email para a lista dizendo que isso foi um engano, e'
 falso...
 
 Eu procuro voce amanha e mostro um contra-exemplo.
 
 A ideia e' comecar em 1 diminuir de 1/2 em 1/2 ate' ficar negativo
  depois cresca de 1/3 em 1/3 ate' passar 1, depois diminuir de 1/4 em 1/4
  ate' ficar negativo, ai' cresce de 1/5 em 1/5 ate'...
 
Essa sequencia tem a propriedade desejada, e todos os pontos do
  intervalo [0,1] sao pontos limite da sequencia.
 
  Valem algumas coisas mais.
 
  Abraco,
 
  Mane'
 
  On Wed, 16 Jul 2003, Salvador Addas Zanata wrote:
 
  
  
   On Wed, 16 Jul 2003 [EMAIL PROTECTED] wrote:
  
Seja x_{k} uma sequencia de numeros reais tal que
   
lim | x_{k+1} - x_{k} | = 0
   
para cada item, demonstre ou dê um contra-exemplo:
   
a) x_{k} é limitada.
  
   Se x_{k}=x_{k-1}+1/k, com x_{0}=0, entao x_{k} nao e limitada.
  
  
b) x_{k} é convergente.
  
  
   Nao eh, pelo exemplo acima.
  
  
c) se x_{k} é limitada então x_{k} é convergente.
   
  
   Isso eh verdade, e so imaginar que se ela nao fosse convergente, teria
 2
   pontos de acumulacao pelo menos e isso implica um absurdo com a sua
   hipotese. Lembre que num compacto, toda seq. tem pontos de acumulacao.
  
  
  
   Abraco,
  
   Salvador
  
  
  
  
  
agradeço qualquer ajuda !
   
   
   
--
Use o melhor sistema de busca da Internet
Radar UOL - http://www.radaruol.com.br
   
   
   
   
 =
Instruções para entrar na lista, sair da lista e usar a lista em
http://www.mat.puc-rio.br/~nicolau/olimp/obm-l.html
   
 =
   
  
  
 =
   Instruções para entrar na lista, sair da lista e usar a lista em
   http://www.mat.puc-rio.br/~nicolau/olimp/obm-l.html
  
 =
  
 
 
 
 =
 Instruções para entrar na lista, sair da lista e usar a lista em
 http://www.mat.puc-rio.br/~nicolau/olimp/obm-l.html
 =
 




-
This mail sent through IMP: http://horde.org/imp/
=
Instruções para entrar na lista, sair da lista e usar a lista em
http://www.mat.puc-rio.br/~nicolau/olimp/obm-l.html
=


[obm-l] Problemas da IMO

2003-07-14 Por tôpico gugu


Prova da IMO retirada do Site http://www.mathlinks.go.ro/

O Problema 1 é nois que mandou...


First Day - 44th IMO 2003 Japan 

1. Let A be a 101-element subset of the set S={1,2,3,...,100}. Prove that 
there exist numbers t_1, t_2, ..., t_{100} in S such that the sets 

Aj = { x + tj | x is in A } for each j = 1, 2, ..., 100 

are pairwise disjoint. 


2. Find all pairs of positive integers (a,b) such that the number 

a^2 / ( 2ab^2-b^3+1) is also a positive integer. 

3. Given is a convex hexagon with the property that the segment connecting the 
middle points of each pair of opposite sides in the hexagon is  sqrt(3) / 2 
times the sum of those sides' sum. 

Prove that the hexagon has all its angles equal to 120. 


Second Day - 44th IMO 2003 Japan 

4. Given is a cyclic quadrilateral ABCD and let P, Q, R be feet of the 
altitudes from D to AB, BC and CA respectively. Prove that if PR = RQ then the 
interior angle bisectors of the angles  ABC and  ADC are concurrent on AC. 

5. Let x1 = x2 = ... = xn be real numbers, n2. 

a) Prove the following inequality: 

(sum  ni,j=1 | xi - xj | ) 2 = 2/3 ( n^2 - 1 )sum ni,j=1 ( xi - xj)^2 

b) Prove that the equality in the inequality above is obtained if and only if 
the sequence (xk) is an arithemetical progression. 

6. Prove that for each given prime p there exists a prime q such that n^p - p 
is not divisible by q for each positive integer n. 



-
This mail sent through IMP: http://horde.org/imp/
=
Instruções para entrar na lista, sair da lista e usar a lista em
http://www.mat.puc-rio.br/~nicolau/olimp/obm-l.html
=


Re: [obm-l] Nenhuma ajuda mesmo: Outro problema

2003-07-11 Por tôpico gugu
Cara Alininha,
   Podemos provar isso por contradicao: se T nao e' continua, existe uma 
 sequencia de vetores v_n em X com ||v_n||=1 e ||T(v_n)||=4^n. Existe tambem, 
 por Hahn-Banach, para cada n=1 um funcional linear f_n em Y* com ||f_n||=1 tal 
 que ||f_n(T(v_n))||=||T(v_n)||=4^n, e, para n=2, podemos escolher esse f_n de 
 modo que f_n(T(v_n)) tenha o mesmo sinal que soma(k=1 a n-1)(f_k(T(v_n))/3^k) 
 (escolhemos no inicio todos os v_n e ja' escolhemos os f_k para 1=k=n-1). 
 Agora consideramos y*=soma(k=1 a infinito)((1/3^k).f_k). Temos ||y*||= 
 =soma(k=1 a infinito)(1/3^k)=1/2, logo y* pertence a Y*. Vamos ver que 
 y*(T(x)):X-R nao e' continua: para isso vamos estimar |y*(T(v_n))|. Como 
 f_n(T(v_n)) tem o mesmo sinal que soma(k=1 a n-1)||f_n(T(v_n))||, temos 
 ||soma(k=1 a n)(f_k(T(v_n))/3^k)||=||f_n(T(v_n))||/3^n. Por outro lado, para 
 cada k, como ||f_k||=1, temos ||f_k(T(v_n))||=||T(v_n)||, donde 
 ||soma(k=n+1 a infinito)(f_k(T(v_n))/3^k)||= 
 = ||T(v_n)||.soma(k=n+1 a infinito)(1/3^k) = ||T(v_n)||/(2.3^n), donde 
 |y*(T(v_n))|= 
 =||soma(k=1 a n)(f_k(T(v_n))/3^k)||-||soma(k=n+1 a
infinito)(f_k(T(v_n))/3^k)|| 
 = ||f_n(T(v_n))||/3^n-||f_n(T(v_n))||/(2.3^n)=||f_n(T(v_n))||/(2.3^n)= 
 =4^n/(2.3^n)=((4/3)^n)/2, pois ||f_n(T(v_n))||=4^n. Portanto, como 
 lim(n-infinito)(((4/3)^n)/2)=infinito, segue que 
 lim(n-infinito)(|y*(T(v_n))|)=infinito, e, logo, como ||v_n||=1 para todo n, 
 temos que y*(T(x)):X-R nao e' limitada, e portanto nao e' continua. 

   Abracos,
Gugu

Quoting Alininha [EMAIL PROTECTED]:

 (...)
 
 Aproveito para perguntar um outro problema que acredito 
 seja bem simples também.
 
 Seja T:X - Y uma aplicação linear ( X é Banach e Y é 
 normado) e Y* o dual de Y. Mostrar que se 
 y*(T(x)):X - R é contínua para cada y* pertencente a Y* 
 então T é contínua.
 
 Mais uma vez Gugu muito obrigada!
 
 


-
This mail sent through IMP: http://horde.org/imp/
=
Instruções para entrar na lista, sair da lista e usar a lista em
http://www.mat.puc-rio.br/~nicolau/olimp/obm-l.html
=


Re: [obm-l] Mol fatorial

2003-07-08 Por tôpico gugu
   E' um programa que faz contas (em particular com numeros grandes e precisao
arbitraria).
   Abracos,
 Gugu

Quoting __GAGO_BOY__ [EMAIL PROTECTED]:

 oi. Gugu, o que é o Mathematica que vc falou?
 
  
 __
 Acabe com aquelas janelinhas que pulam na sua tela.
 AntiPop-up UOL - É grátis!
 http://antipopup.uol.com.br/
 
 
 =
 Instruções para entrar na lista, sair da lista e usar a lista em
 http://www.mat.puc-rio.br/~nicolau/olimp/obm-l.html
 =
 




-
This mail sent through IMP: http://horde.org/imp/
=
Instruções para entrar na lista, sair da lista e usar a lista em
http://www.mat.puc-rio.br/~nicolau/olimp/obm-l.html
=


Re: [obm-l] Nenhuma ajuda mesmo

2003-07-08 Por tôpico gugu
   Cara Alininha,
   Contei esse problema para o Artur Avila, que deu a seguinte solucao: ele
considerou exatamente os seus conjuntos C e D, e assim o teorema de Hahn-Banach
na forma da separacao (qual o enunciado que voce tem dessa forma do teorema de
Hahn-Banach ?) implica a existencia de um funcional linear continuo g(x,t)
definido em X x R tal que g(x,t)0 para todo (x,t) em D e g(x,y)=0 para todo
(x,t) em C. Por outro lado, g(x,t)=h(x)+bt, para algum b real nao nulo (se b
fosse 0 teriamos que g(a,t)=h(a) nao dependeria de t, mas se a esta'  em A e t
e'  muito grande entao (a,t) esta'  em D, e, se t e'  muito negativo, (a,t)
esta' em C, absurdo, pois C nao intersecta D, por hipotese). Assim, para todo a
em A, h(a)+b.f(a) = 0, pois (a,f(a)) pertence a C, e para todo x em X e todo t 
 M||x|| temos h(x)+b.t  0, pois nesse caso (x,t) pertence a D.b Em particular
b deve ser maior que 0 (senao h(x)+bt ficaria negativo para t muito grande). Da
primeira desigualdade, segue entao que f(a)=-h(a)/b, e da segunda (fazendo t
tender por cima a M||x||) segue que h(x)+b.M.||x|| = 0, donde -h(x)/b=M||x||,
e portanto, se definirmos x*(x)=-h(x)/b, temos que x* e'  um funcional linear em
X* com||x*||=M tal que f(a)=x*(a) para todo a em A, cqd.
Abracos,
 Gugu

Quoting Alininha [EMAIL PROTECTED]:

 Obrigada Gugu por tentar me ajudar.
 
 Acho que misturei um pouco o enunciado com a minha 
 tentativa de solução.
 Estava tentando aplicar Hahn-Banhach na forma da 
 separação e para isso eu defini os conjuntos:
 
 C= { (a,t) tal que a pertence a a e f(a)= t}
 D= { (x,t) tal que x pertence a X e M||x||t}
 
 Onde montrei que C é fechado e convexo, D é aberto e 
 convexo e que C intersecção com D é vazio. Estando assim 
 em condições de aplicar Hahn-Banach na forma da 
 separação o que não consegui. Abaixo reescrevo o 
 enunciado do problema.
 
 X é um espaço normado REAL, A é um subconjunto convexo 
 de X com o elemento neutro de X pertencente a A. 
 Consideremos ainda uma função côncava f 
 satisfazendo f(a) = M ||a|| para todo a em A (||a|| é 
 norma de a).
 Queremos mostrar que existe um elemento x* do dual de X 
 tal que ||x*||=M e f(a)=x*(a) com a de A e x de X.
 
 Aproveito para perguntar um outro problema que acredito 
 seja bem simples também.
 
 Seja T:X - Y uma aplicação linear ( X é Banach e Y é 
 normado) e Y* o dual de Y. Mostrar que se 
 y*(T(x)):X - R é contínua para cada y* pertencente a Y* 
 então T é contínua.
 
 Mais uma vez Gugu muito obrigada!
 
 
 Cara Alininha,
 Na verdade eu acho que nao entendi bem o seu enuncia
 do: Voce usa o nome A
  para dois conjuntos: o subconjunto convexo de X dado in
 icialmente e 
  A= {(a,t) tal que a pertence a a e f(a)
 = t}. Por outro lado, voce define o
  conjunto B mas depois nao fala mais nele... A qual conj
 unto A se refere a
  ultima frase ? Que papel tem o conjunto B no problema ?
 Abracos,
  Gugu
  
  
  Sei que o problema é um pouco off-topic mas aqui me 
  parece o único lugar onde posso obter ajuda para os me
 us 
  estudos.
  
  Qualquer ajuda para resolver o problema abaixo será 
  excelente. Já esgotei meu conhecimento.
  
  ---
  Abaixo repito o problema
  ---
  Acredito que seja uma aplicação imediata do Teorema de
  
  Hahn-Banach na forma da separação, entretanto, como 
  surge um produto cartesiano de dois espaços não conseg
 ui 
  (para minha tristeza) escrever a solução.
  O problema é o seguinte: 
  
  
  X é um espaço normado REAL, A 
  é um subconjunto convexo de X com o elemento neutro de
  X 
  pertencente a A. Consideremos ainda uma função côncava
  f 
  satisfazendo f
 (a) = M ||a|| para todo a em A (||a|| é 
  norma de a) e os subconjunto do produto cartesiano de 
 X 
  com R:
  A= { (a,t) tal que a pertence a a e f(a)= t}
  B= { (x,t) tal que x pertence a X e M||x||t}
  
  Queremos mostrar que existe um elemento x* do dual de 
 X 
  tal que ||x*||=M e f(a)=x*(a) com a de A e x de X.
  
  Serei muito grata pela ajuda.
  
  Alininha
  
  
   
  __
 
  Seleção de Softwares UOL.
  10 softwares escolhidos pelo UOL para você e sua famíl
 ia.
  http://www.uol.com.br/selecao
  
  
  ==
 ===
  Instruções para entrar na lista, sair da lista e usar 
 a lista em
  http://www.mat.puc-rio.br/~nicolau/olimp/obm-l.html
  ==
 ===
  
  ===
 ==
  Instruções para entrar na lista, sair da lista e usar a
  lista em
  http://www.mat.puc-rio.br/~nicolau/olimp/obm-l.html
  ===
 ==
  
 
  
 __
 Seleção de Softwares UOL.
 10 softwares escolhidos pelo UOL para você e sua família.
 http://www.uol.com.br/selecao

Re: [obm-l] off: IMPA

2003-06-19 Por tôpico gugu
   Caro Morgado,
   Eu queria agradecer as referencias (um tanto exageradas) a meu respeito. 
Eu queria aproveitar para complementar minha mensagem anterior sobre o assunto, 
pois me perguntaram (fora da lista) se era necessario ter o curriculo avaliado 
para fazer cursos no IMPA, e achei que isso poderia interessar a mais gente da 
lista. Qualquer curso de matematica do IMPA (de iniciacao cientifica, mestrado 
ou doutorado) pode ser feito como curso livre, gratuitamente (as inscricoes sao 
feitas ate' 10 dias depois do inicio do curso). Para pedir bolsa de mestrado ou 
de doutorado (ou bolsa de verao, para alunos de fora do Rio fazerem cursos no 
verao) sao analisados historicos escolares e cartas de recomendacao. Vejam a 
pagina do IMPA (www.impa.br) para mais informacoes.  
   Abracos,
Gugu

Citando A. C. Morgado [EMAIL PROTECTED]:

 Diego,
 Gugu, ou melhor, o Professor Doutor Carlos Gustavo Tamm de Araújo 
 Moreira, pesquisador do IMPA, parceiro do Yoccoz (medalha Fields) em 
 varios trabalhos, que nos honra dedicando parte de seu tempo a essa 
 lista e a preparaçao dos nossos olimpicos, terminou o mestrado antes de 
 terminar o Santo Inacio. Todos os que tiveram desempenho excepcional em 
 Olimpiadas, se se dedicaram a Matematica, tornaram-se brilhantes 
 pesquisadores, tanto no Brasil como no mundo.
 O que talvez seja curioso eh que a porcentagem de olimpicos brilhantes 
 que se dedicaram a Matematica nao eh tao alta como se poderia esperar. 
 Eu penso, no olhometro, que deva ser de no maximo 40%. Nos EUA e Canada, 
 ha uma competiçao entre universitarios do ciclo basico (a Putnam). 
 Somente cerca de 30% dos vencedores da Putnam tornaram-se matematicos. 
 Mas alguns dos mais importantes fisicos, quimicos, advogados e medicos 
 dos EUA foram vencedores da Putnam, ateh um Nobel de Medicina.
 Alguem ja disse, e disse muito bem, que um bom sistema educacional eh o 
 que permite a um Mozart tornar-se um Mozart. No Brasil, Mozart 
 dificilmente se tornaria um Mozart. Seria reprovado em Quimica , nao 
 terminaria o ensino medio e nao entraria para a Escola de Musica ( o 
 grande pianista Jacques Klein nao pode ensinar piano na Escola de Musica 
 por nao ter curso superior). Ainda bem que o IMPA eh assim.
 Abraços.
 Morgado
 
 
 Diego Navarro wrote:
 
 Eu só não entendo como o IMPA matricula no mestrado alguém sem segundo grau
 completo. Sim,
 pode ser um gênio espontâneo à Ramanujan - ou pode ser um aleatório cortando
 caminho. Não
 me parece que desempenho extraordinário em olimpíadas tenha algo a ver com
 pesquisa em
 nível de doutorado.
 
 Daqui a pouco o Nicolau me repreende por continuar a thread off-topic. Tudo
 bem, eu
 mereço.
 - Original Message -
 From: [EMAIL PROTECTED]
 To: [EMAIL PROTECTED]
 Sent: Tuesday, June 17, 2003 6:23 PM
 Subject: Re: [obm-l] off: IMPA
 
 
   
 
Na verdade eu acho que o Niski se refere ao caso do Leonardo Macarini,
 que
 saiu do colegio no primeiro grau (talvez por acha-lo chato), e depois de
 algum
 tempo foi estudar no IMPA, onde fez o mestrado e o doutorado (o qual acabou
 com
 uns 23 ou 24 anos, ou seja, nao tao jovem assim). O conselho federal de
 educacao acabou outorgando a ele os diplomas anteriores (do primeiro grau
 a
 graduacao) e validando seu doutorado. Atualmente ele e' pesquisador
 associado
 do IMPA (com uma bolsa Profix).
Abracos,
 Gugu
 
 
 Quoting Nicolau C. Saldanha [EMAIL PROTECTED]:
 
 
 
 On Tue, Jun 17, 2003 at 09:32:41AM -0700, niski wrote:
   
 
 Alguem sabe se é verdade que um garoto no IMPA recebeu no mesmo dia os
 diplomas de graduacao,mestrado e doutorado?!
 
 
 Não sei, mas não é tão improvável.
 
 Tem muita gente que começa a pós antes de se formar, isso é comum.
 Alguns começam a achar a graduação chata e abandonam, o que não é
 recomendável.
 Mesmo assim o cara pode completar as exigências para mestrado e
 doutorado
 mas não pode receber o diploma pq não tem o diploma de graduação.
 Aí se um dia o cara completar a graduação ele pode receber os diplomas
 de mestrado e doutorado sem precisar fazer mais nada (ele já fez
 o que precisava) mas dificilmente os diplomas ficarão prontos
 instantaneamente.
 
 []s, N.
 =
 Instruções para entrar na lista, sair da lista e usar a lista em
 http://www.mat.puc-rio.br/~nicolau/olimp/obm-l.html
 =
 
   
 
 
 
 -
 This mail sent through IMP: http://horde.org/imp/
 =
 Instruções para entrar na lista, sair da lista e usar a lista em
 http://www.mat.puc-rio.br/~nicolau/olimp/obm-l.html
 =
 
 
 
 
 =
 Instruções para entrar na lista, sair da

[obm-l] Re:

2003-06-19 Por tôpico gugu
   Caro Helder,
 (a)  Faltou ABB na sua lista, mas acho que foi por esquecimento, nao ?
 Nesse caso a resposta e' 10: AAABBBABAA, por exemplo (como sao 8 sequencias, 
que tem 8 primeiras letras e' claro que 10 e' o menor possivel; se fosse num 
circulo e nao numa fila a resposta seria 8). Se voce quiser mesmo omitir o ABB 
a resposta passa a ser 9: BBBAAABAB, por exemplo.
 (b) A resposta e' 2^n+n-1 (que e' claramente o minimo possivel, como acima; se 
for num circulo a resposta e' 2^n). Ha' mais de uma prova interessante desse 
fato (essas sequencias se chamam sequencias de de Brujin, se nao me engano). 
Acho que vou deixar que as outras pessoas pensem um pouco sobre como provar 
este fato, e depois, se for necessario, eu escrevo uma prova.
   Abracos,
Gugu

Citando Helder Suzuki [EMAIL PROTECTED]:

 Se eu tenho muitos carros azuis ou brancos, e eu faço
 uma fila com somente 3 desses carros, posso ter uma
 das seguintes combinações:
 AAA
 AAB
 ABA
 BAA
 BAB
 BBA
 BBB
 onde A indica um carro azul e B indica um carro
 branco.
 
 (a) Qual a quantidade mínima de carros(azuis e
 brancos) que eu preciso para formar uma única fila tal
 que eu possa encontrar dentro dela todas as
 combinações de 3 carros?
 obs: as combinações podem se sobrepor
 
 (b) Qual a quantidade mínima de carros(azuis e
 brancos) que eu preciso para formar uma única fila tal
 que eu possa encontrar dentro dela todas as
 combinações de N carros?
 obs: as combinações podem se sobrepor
 
 Abraços,
 Helder Toshiro Suzuki
 
 ___
 Yahoo! Mail
 Mais espaço, mais segurança e gratuito: caixa postal de 6MB, antivírus,
 proteção contra spam.
 http://br.mail.yahoo.com/
 =
 Instruções para entrar na lista, sair da lista e usar a lista em
 http://www.mat.puc-rio.br/~nicolau/olimp/obm-l.html
 =
 




-
This mail sent through IMP: http://horde.org/imp/
=
Instruções para entrar na lista, sair da lista e usar a lista em
http://www.mat.puc-rio.br/~nicolau/olimp/obm-l.html
=


Re: [obm-l] off: IMPA

2003-06-17 Por tôpico gugu
   Na verdade eu acho que o Niski se refere ao caso do Leonardo Macarini, que 
saiu do colegio no primeiro grau (talvez por acha-lo chato), e depois de algum 
tempo foi estudar no IMPA, onde fez o mestrado e o doutorado (o qual acabou com 
uns 23 ou 24 anos, ou seja, nao tao jovem assim). O conselho federal de 
educacao acabou outorgando a ele os diplomas anteriores (do primeiro grau a 
graduacao) e validando seu doutorado. Atualmente ele e' pesquisador associado 
do IMPA (com uma bolsa Profix).
   Abracos,
Gugu
 

Quoting Nicolau C. Saldanha [EMAIL PROTECTED]:

 On Tue, Jun 17, 2003 at 09:32:41AM -0700, niski wrote:
  Alguem sabe se é verdade que um garoto no IMPA recebeu no mesmo dia os 
  diplomas de graduacao,mestrado e doutorado?!
 
 Não sei, mas não é tão improvável.
 
 Tem muita gente que começa a pós antes de se formar, isso é comum.
 Alguns começam a achar a graduação chata e abandonam, o que não é
 recomendável.
 Mesmo assim o cara pode completar as exigências para mestrado e doutorado
 mas não pode receber o diploma pq não tem o diploma de graduação.
 Aí se um dia o cara completar a graduação ele pode receber os diplomas
 de mestrado e doutorado sem precisar fazer mais nada (ele já fez
 o que precisava) mas dificilmente os diplomas ficarão prontos
 instantaneamente.
 
 []s, N.
 =
 Instruções para entrar na lista, sair da lista e usar a lista em
 http://www.mat.puc-rio.br/~nicolau/olimp/obm-l.html
 =
 




-
This mail sent through IMP: http://horde.org/imp/
=
Instruções para entrar na lista, sair da lista e usar a lista em
http://www.mat.puc-rio.br/~nicolau/olimp/obm-l.html
=


Re: [obm-l] Número neperiano

2002-04-19 Por tôpico gugu

Eu costumo contar uma estorinha do seguinte tipo:suponha que um sujeito
empreste 1 real a outro a taxa de 100% ao ano.No fim do ano o outro vem com 2
reais para pagar a divida.O primeiro argumenta:veja bem,na metade do ano voce
ja'  me devia 1,50,e na outra metade incidem mais 50% sobre os 1,50,donde voce
me deve pelo menos 2,25.Por outro lado ele podia dividir o ano em 10 pedacos e
aplicar em cada um taxa de 10%,o que daria (1,1)^10=2,59374246,que e' mais que
2,25.Se dividirmos o ano em n partes iguais o valor fica (1+1/n)^n.
O maximo que o primeiro sujeito consegue cobrar com essa conversa fiada e'
e=2,718281828459045235360287...=lim(1+1/n)^n,o que corresponde a dividir o ano
em infinitos pedacos.
Abracos,
Gugu


Quoting Bruno F. C. Leite [EMAIL PROTECTED]:

 At 02:18 19/04/02 -0300, you wrote:
 
  Como se explica o que é número neperiano p/um aluno do 3º ano do
 Ensino Médio (ou seja, um vrestibulando)? Vale lembrar que o sujeito NÃO
 está familiarizado com log e NUNCA viu exp... Obviamente, dizer q é a
 base de log resultante da integral de 1/x tb não vale :0)
 
 []'s
 
 Você está pedindo uma definição de E que não use log ou exp? Se sim, 
 pode-se usar
 E=soma(1/k!),k=0,1,2..
 
 ou
 
 E=limite(1+1/n)^n
 
 Tudo bem que as duas envolvem limites, mas é mais ou menos intuitivo para 
 um vestibulando (ele já viu a soma dasérie geométrica). Dá para mostrar 
 elementarmente que a série de 1/K! converge, pq k!2^k para k=4.
 
 Bruno Leite
 http://www.ime.usp.br/~brleite
 
 
 
 Alexandre Tessarollo
 
 =
 Instruções para entrar na lista, sair da lista e usar a lista em
 http://www.mat.puc-rio.br/~nicolau/olimp/obm-l.html
 O administrador desta lista é [EMAIL PROTECTED]
 =
 
 =
 Instruções para entrar na lista, sair da lista e usar a lista em
 http://www.mat.puc-rio.br/~nicolau/olimp/obm-l.html
 O administrador desta lista é [EMAIL PROTECTED]
 =
 




-
This mail sent through IMP: http://horde.org/imp/
=
Instruções para entrar na lista, sair da lista e usar a lista em
http://www.mat.puc-rio.br/~nicolau/olimp/obm-l.html
O administrador desta lista é [EMAIL PROTECTED]
=